Hand Nerves Flashcards

1
Q

A 30-year-old man is brought to the emergency department after a rollover motor vehicle collision. Physical examination shows significant soft-tissue loss and a median nerve injury. At the time of surgical repair, there is a 5-cm gap in the nerve. Which of the following treatment options is most likely to provide the best long-term functional outcome for this patient?

A) Acellular autograft
B) Autologous vein graft
C) Collagen conduit
D) Peripheral nerve autograft
E) Polyglycolic acid conduit

A

The correct response is Option D.

For bridging long nerve gaps not amenable to primary repair (greater than 3 cm), peripheral nerve autografts are the most reliable choice. Their use is limited by supply and donor site morbidity from additional incisions, loss of sensation, and possible neuromas. Common donor sites include sural nerve, medial antebrachial cutaneous nerve, lateral antebrachial cutaneous nerve, dorsal cutaneous branch of the ulnar nerve, superficial peroneal nerve, and posterior and lateral cutaneous nerves of the thigh.

Nerve conduits, such as silicone tubes, synthetic biodegradable tubes (such as polyglycolic acid conduit or collagen conduit), and biologic tubes (such as autologous vein grafts) are limited to use in short gaps (less than 3 cm). Beyond 3 cm, there is no clinically meaningful regeneration. Acellular autografts have recently been used with good success in short nerve gaps (less than 3 cm). The advantage of these products is that they provide the extracellular matrix molecules, which may enhance nerve regeneration.

2018

How well did you know this?
1
Not at all
2
3
4
5
Perfectly
2
Q

A 50-year-old male construction worker is evaluated for weakness of grip and pinch with inability to touch index finger to thumb. The patient is referred from a neurologist with a diagnosis of anterior interosseous nerve syndrome. Which of the following muscles is most likely to be spared?

A) Flexor digitorum profundus to index
B) Flexor digitorum profundus to middle
C) Flexor pollicis longus
D) Pronator quadratus
E) Pronator teres

A

The correct response is Option E.

The pronator teres is innervated by the median nerve prior to its take off of the anterior interosseous nerve (AIN), which is why it cannot be affected by AIN syndrome. Pronation generated by the pronator teres or the pronator quadratus would be indistinguishable clinically. The AIN is a terminal branch off the median nerve that innervates the flexor digitorum profundus to the index and middle fingers, flexor pollicis longus (FPL) and pronator quadratus. The AIN arises from the median nerve approximately 4 to 6 cm distal to the medial epicondyle. It travels between the flexor digitorum profundus (FDP) and flexor digitorum superficialis (FDS) initially, and then between the FPL and FDP. Then it lies on the volar surface of the interosseous membrane and travels with the anterior interosseous artery, terminating in the pronator quadratus and then the wrist joint capsule and the intercarpal, radiocarpal, and radioulnar joints. The nerve originates from C5-T12, becoming the medial and lateral cords of the brachial plexus and then becoming the median nerve.

2018

How well did you know this?
1
Not at all
2
3
4
5
Perfectly
3
Q

Which of the following is the type of axon fiber (neuron) that is primarily involved with the autonomic changes that occur with complex regional pain syndrome (CRPS) type 1?

A) A delta sensory
B) Alpha motor
C) C sensory
D) Gamma motor
E) Ia sensory

A

The correct response is Option C.

C sensory fibers are responsible for a deeper, more non-localizable pain. C fibers can react to various stimuli, including thermal, mechanical, or chemical. C fibers respond to physiologic changes in the body, such as hypoxia, hypoglycemia, hypo-osmolarity, the presence of muscle metabolic products, and light or sensitive touch.

Paul Sudeck noticed that CRPS demonstrates classic inflammatory signs such as pain, swelling, erythema, hyperthermia, and impaired function. However, clinical chemistry markers of inflammation are not elevated. These findings imply a neurogenic inflammation. C fibers have an afferent function in the mediation of pain (and itch), but also an efferent neurosecretory function. They release neuropeptides such as substance P and calcitonin-gene-related peptide (CGRP). The presence of these neuropeptides might explain trophic and autonomic symptoms such as swelling, erythema, and hyperhidrosis. Elevated CGRP levels are also associated with autonomic disturbances, mainly with hyperhidrosis. Also, a role for CGRP in hair growth is suggested, and substance P seems to be involved in the regulation of osteoclastic activity.

Alpha motoneurons innervate muscle fibers of skeletal muscle and are directly responsible for initiating their contraction. When the central nervous system sends out signals to alpha neurons to fire, signals are also sent to gamma motoneurons to do the same. This process maintains the tautness of muscle spindles and is called alpha gamma co-activation. Without gamma motoneurons, muscle spindles would be very loose as the muscle contracts. Unrestricted alpha activity would not allow for muscle spindles to detect a precise amount of stretch and would not allow for optimization of muscle function.

Ia sensory fibers are a type of proprioceptor that is found inside the muscle itself. They lie parallel to the contractile fibers, and give them the ability to precisely monitor muscle length.

A delta fiber is a type of sensory nerve fiber. A delta fibers carry cold, pressure and some pain signals. Because A delta fibers have a higher conduction velocity, and are responsible for quick, shallow pain to a specific area. They are activated by a stimulus of weaker intensity, and are not responsible for the autonomic changes seen with CRPS.

2018

How well did you know this?
1
Not at all
2
3
4
5
Perfectly
4
Q

A 24-year-old man comes to the office because of a brachial plexus injury sustained in a motorcycle accident. Nerve transfer to the biceps for restoration of elbow flexion is planned. Which of the following fascicles or nerves is the most appropriate donor for the transfer?

A) Distal spinal accessory nerve
B) Flexor carpi ulnaris fascicle of the ulnar nerve
C) Medial pectoral nerve
D) Palmaris longus fascicle of the median nerve
E) Thoracodorsal nerve

A

The correct response is Option B.

Adult upper trunk brachial plexus injuries result in significant disability. Several surgical treatment strategies exist, including nerve grafting, nerve transfers, and a combination of both approaches.

The flexor carpi ulnaris (FCU) fascicle of the ulnar nerve to biceps transfer was first described by Oberlin et al in 1994. Generally, the donor nerve with the largest caliber and the greatest number of motor axons should be used for elbow flexion. The other suggested nerve transfer options are also possibilities, but are not as preferable as the FCU fascicle transfer. MacKinnon has advocated transfer of the FCU fascicle of the ulnar nerve to the biceps and FCR fascicle of the median nerve to the brachialis to maximize recovery of elbow flexion.

2018

How well did you know this?
1
Not at all
2
3
4
5
Perfectly
5
Q

A 22-year-old man comes to the emergency department after sustaining a laceration to the dorsal thumb by punching a glass door. Radial nerve block is planned during surgical repair. Which of the following is the approximate distance proximal to the radial styloid in which the superficial branch of the radial nerve pierces the deep fascia?

A) 0 to 4 cm
B) 5 to 9 cm
C) 10 to 14 cm
D) 15 to 19 cm

A

The correct response is Option B.

The superficial branch of the radial nerve runs below the brachioradialis muscle in the mid-forearm, later becoming sub-fascial between the brachioradialis and extensor carpi radialis longus (ECRL) muscles. Approximately 8 to 9 cm proximal to the radial styloid, the superficial branch of the radial nerve (SBRN) becomes subcutaneous, piercing the fascia. The ideal location of infiltration for a radial nerve block is at the sub-fascial location just before the nerve becomes subcutaneous.

2018

How well did you know this?
1
Not at all
2
3
4
5
Perfectly
6
Q

A 53-year-old woman is evaluated for a 1-year history of numbness and tingling of the thumb and the index and long fingers of the right hand. She has been unresponsive to conservative treatment. An increase in which of the following is most likely suggestive of carpal tunnel syndrome in this patient?

A) Abductor digiti minimi fibrillations
B) Adductor pollicis fibrillations
C) Motor nerve conduction velocity
D) Sensory distal latency
E) Sensory nerve conduction velocity

A

The correct response is Option D.

The diagnosis of carpal tunnel syndrome is primarily a clinical diagnosis; however, electrodiagnostic studies (EDX) may be helpful in confirming the diagnosis. While these EDX studies are commonly referred to as “EMGs,”’ they are actually two separate studies: the nerve conduction studies (NCS) and the electromyography (EMG). NCS examine both the sensory and the motor nerve fibers. Sensory nerve conduction studies measure sensory nerve action potential, and the motor nerve conduction studies evaluate a compound muscle action potential. The NCS also measures the amplitude of both the compound muscle action potential and sensory nerve action potential. Nerve conduction velocity (NCV), the velocity of the nerve’s action potential between two points, is also measured by the nerve conduction studies.

The EMG tests the muscle itself. The needle electrode examination can measure motor unit potential (MUP). MUP is measured in regards to its amplitude, duration, wave shape, and firing pattern. In the diagnosis of carpal tunnel syndrome, particular attention is given to the MUP of the abductor pollicis brevis muscle, which is uniquely innervated by the median nerve after it passes through the carpal tunnel.

In the diagnosis of carpal tunnel syndrome, changes in the sensory nerve are detected earlier in the carpal tunnel process than motor changes. Early NCS changes (as compared to standardized normal values) include prolonged or increased sensory distal latencies. Prolonged motor latencies (also abnormal) are detected less frequently than the sensory latency changes, and detected in only 35 to 50% of patients with carpal tunnel syndrome. Motor amplitude change, found in carpal tunnel syndrome, is detected even less commonly. A conduction block, or slowing of the nerve’s action potential velocity (NCV) between two points, can be seen with carpal tunnel syndrome.

The needle electrode examination (EMG) is normal in more than 60% of patients with the diagnosis of carpal tunnel syndrome. Fibrillations in the abductor pollicis brevis occurs in generally less than 20% of patients with carpal tunnel syndrome. The adductor pollicis brevis and abductor digiti minimi muscles are innervated by the ulnar nerve, and would not show any electrodiagnostic evidence of muscle instability in isolated carpal tunnel syndrome.

2018

How well did you know this?
1
Not at all
2
3
4
5
Perfectly
7
Q

A 47-year-old man is brought to the emergency department after sustaining a stab wound injury to the left shoulder. Physical examination shows isolated loss of deltoid function. The injured nerve is supplied by which of the following nerve roots?

A) C5 through C6
B) C6 through C7
C) C7 only
D) C7 through T1
E) T1 only

A

The correct response is Option A.

The deltoid muscle receives motor innervation from the axillary nerve. The axillary nerve receives its contributions from C5 and C6 roots. These roots come together to form the superior trunk, which splits into anterior and posterior divisions. The axons heading to the axillary nerve travel in the posterior division, which joins the other posterior divisions from middle and inferior trunks to form the posterior cord. The axillary nerve arises from the posterior cord and travels laterally to innervate the deltoid muscle.

C7 is the primary innervation to the latissimus dorsi and triceps, and contributes to digital extension as well. C8 and T1 primarily serve the hand, providing intrinsic muscle innervation.

2018

How well did you know this?
1
Not at all
2
3
4
5
Perfectly
8
Q

A patient with severe traumatic brachial plexus root avulsion injury is scheduled to undergo functioning free muscle transfer for simultaneous restoration of both elbow flexion and finger flexion, in addition to other reconstructive procedures. Use of which of the following muscles is most appropriate for this purpose?

A) Gracilis
B) Pectoralis major
C) Rectus abdominus
D) Serratus anterior
E) Trapezius

A

The correct response is Option A.

The gracilis muscle is the most commonly described muscle for use as a free functioning muscle in reconstruction of upper extremity function following brachial plexus injury. Common options for use in these reconstructions include the gracilis, latissimus dorsi, rectus femoris, and vastus lateralis. The gracilis muscle has good excursion, size, and length, but does lack strength compared with some other muscle options.

The rectus abdominis, serratus anterior, and trapezius muscles have not been described for free functioning muscle transfer in the upper extremity. Though the pectoralis major muscle was described as a free functioning muscle transfer by Manktelow and McKee in 1978, it has not been a commonly used muscle.

2018

How well did you know this?
1
Not at all
2
3
4
5
Perfectly
9
Q

An 8-year-old boy is brought to the emergency department after sustaining injury to the right upper extremity, 3-cm proximal to the antecubital fossa. Which of the following factors is associated with improved functional outcomes following peripheral nerve repair?

A) Fewer suture strands used in the nerve repair
B) Higher-tension nerve repair
C) Increasing time between nerve injury and repair
D) More proximal nerve injury
E) Younger patient age

A

The correct response is Option E.

The repair of peripheral nerve injuries can be affected by several factors. Younger patients tend to have improved outcomes compared with older patients. Although there is no consensus on the optimal timing for nerve repair, earlier repairs have been shown to have better outcomes than those attempted at later time points. The level at which the injury has occurred can also affect the outcome. The more proximal the injury, the worse the prognosis in terms of motor and sensory return. Moreover, more complete and rapid regain of function occurs in more proximally innervated muscles. Finally, technical aspects of the nerve repair can also affect outcomes. Minimal tension and an increasing number of suture strands crossing the repair site are both associated with improved function.

2017

How well did you know this?
1
Not at all
2
3
4
5
Perfectly
10
Q

A 25-year-old man comes to the office after sustaining a deep laceration to the elbow. Physical examination shows decreased function of the ulnar nerve, and the patient is taken for operative exploration and repair. Following proximal and distal dissection, a 1-cm gap between the proximal and distal nerve ends persists. Which of the following is the most appropriate next step in management?

A) Nerve transfer
B) Nerve transposition
C) Polyglycolic acid nerve conduit
D) Primary repair
E) Sural nerve grafting

A

The correct response is Option B.

Principles of microsurgical nerve repair include the use of meticulous and atraumatic technique with adequate magnification, microsurgical instruments, and sutures. A primary repair is performed whenever possible, provided that the repair is tension-free in order to maximize perfusion to the repair site.

In this patient, a 1-cm nerve gap in the ulnar nerve was present even after mobilizing the proximal and distal nerve ends. In this situation, the ulnar nerve may be transposed anteriorly, which would shorten the distance between the nerve ends and allow for primary repair.

Nerve transfers are indicated in very proximal nerve injuries where a proximal stump is unavailable for primary repair or grafting, or when a very long nerve gap is present where there would be a concern that target muscle denervation might occur prior to nerve regeneration.

Polyglycolic acid nerve conduits are bioabsorbable tubes through which nerve regeneration occurs. They represent an option for nerve reconstruction without any associated donor site morbidity when a nerve gap is present in order to achieve a tension-free repair.

Although primary nerve repair is preferable to the use of a graft/conduit, doing so in the setting of this patient’s 1-cm nerve gap would not result in a tension-free repair.

Autologous nerve grafting, such as with the sural nerve, is an option for nerve reconstruction when a nerve gap is present in order to achieve a tension-free repair.

2017

How well did you know this?
1
Not at all
2
3
4
5
Perfectly
11
Q

A 33-year-old woman comes to the office because of volar numbness of the right thumb, index, long finger, and palm for the past 4 months. Medical history includes no abnormalities. Physical examination shows weakness of the palmar abduction of the thumb and interphalangeal joint flexion of the thumb. Percussion tenderness over the volar aspect of the wrist does not reproduce symptoms. Passive flexion of the wrist does not reproduce symptoms. Which of the following is the most likely diagnosis?

A) Anterior interosseous syndrome
B) Carpal tunnel syndrome
C) Cubital tunnel syndrome
D) Posterior interosseous syndrome
E) Pronator syndrome

A

The correct response is Option E.

Pronator syndrome is a compression neuropathy of the median nerve in the proximal forearm. Paresthesias in the palm and the median nerve–innervated fingers of the thumb along with weakness of the flexor pollicis longus muscle are classic findings in pronator syndrome. Pronator syndrome can be differentiated from carpal tunnel syndrome by sensory symptoms and numbness occurring in the palm, which is innervated by the palmar cutaneous branch of the median nerve. This branch comes off the proper median nerve proximal to the carpal tunnel. Findings of weakness of the flexor pollicis longus muscle and often the flexor digitorum profundus muscle to the index finger are often seen in pronator syndrome. Anterior interosseous syndrome is a motor-only compression of the median nerve in the proximal forearm; sensory findings are not present. Cubital tunnel syndrome or ulnar nerve compression at the elbow presents with sensory findings in the small and ring fingers and may also affect the interosseous muscles. Posterior interosseous syndrome affects the extensor muscles in the forearm and is not the pathologic lesion described in the scenario.

2017

How well did you know this?
1
Not at all
2
3
4
5
Perfectly
12
Q

A 30-year-old woman comes to the office because of burning pain in the left wrist 5 months after discharge from the hospital. During her visit, a catheter was inserted in the left radial artery for arterial line monitoring in the intensive care unit. On examination, point tenderness and Tinel sign are noted over the volar radial aspect of the forearm, just ulnar to the radial artery, and overlying the flexor carpi radialis tendon. Which of the following is the most likely nerve of origin for the suspected condition?

A) Anterior interosseous
B) Median
C) Musculocutaneous
D) Radial
E) Ulnar

A

The correct response is Option C.

The neuroma is of the lateral antebrachial cutaneous nerve, which innervates the area in question, over the volar forearm, including the skin overlying the flexor carpi radialis (FCR) tendon. The lateral antebrachial cutaneous nerve is the continuation of the musculocutaneous nerve in the forearm.

The dorsal radial sensory nerve innervates the skin overlying the dorsal and radial aspects of the wrist and does not innervate the skin overlying the FCR tendon. Therefore, the radial nerve is not appropriate.

The ulnar and median nerves are not appropriate, as they give off sensory contributions in the palm and fingers. The anterior interosseous nerve is mainly a motor nerve in the forearm, finally sending off branches to the volar capsule of the wrist.

2017

How well did you know this?
1
Not at all
2
3
4
5
Perfectly
13
Q

A 29-year-old man is brought to the emergency department because of a deep laceration of the medial right arm at the elbow. Examination shows complete loss of ulnar nerve function to the right hand. During exploration in the operating room, complete transection of the ulnar nerve at the elbow with a 3-cm nerve gap after debridement is noted. Which of the following procedures is most likely to result in the earliest recovery of intrinsic muscle function in this patient’s hand?

A) Anterior interosseous nerve transfer
B) Cable grafting with nerve allografting
C) Cable grafting with sural nerve grafting
D) Primary repair with the elbow in flexion
E) Ulnar nerve transposition and primary repair

A

The correct response is Option A.

The most appropriate answer is transfer of the anterior interosseous nerve to the motor branch of the ulnar nerve in the distal forearm. Ulnar nerve injuries are especially debilitating with loss of grip and claw hand deformity. High ulnar nerve injuries are considered to be anything proximal to the innervation of the flexor carpi ulnaris and flexor digitorum profundus muscles at or near the elbow. This includes the elbow, upper arm, and brachial plexus.

The prognosis for recovery of intrinsic hand function is poor in high ulnar nerve injuries 30 to 35 cm proximal to the hypothenar eminence. This is due to the length of time required for nerve regeneration, and the motor endplate degeneration that occurs during this time. High ulnar nerve injuries will exceed the approximately 18-month window for regeneration in order to achieve meaningful muscle recovery.

The theory behind nerve transfer surgery is to take an expendable donor nerve and use the fascicles to restore function to a more critically injured nerve. The anastamosis for an AIN-to-ulnar nerve transfer is 8 to 10 cm proximal to the wrist crease and greatly decreases the amount of distance and therefore time required for reinnervation of intrinsic hand muscles. Although the anterior interosseous nerve contains 75% of the axons of the deep motor branch of the ulnar nerve, meaningful recovery of intrinsic muscle function can be expected. The anastamosis for the transfer can be done in an end:end or end:side technique. End-to-side nerve transfer is indicated in partial nerve injuries or lower nerve injuries where primary repair of the injured nerve is possible and one can expect some contribution of nerve fibers from the native nerve.

Primary repair of any nerve injury under tension or that requires extreme joint flexion is not indicated. This results in tension at the repair site, internal scarring, possible flexion contracture, and a poor result. Anterior transposition of the ulnar nerve may be performed in some instances but is reported to gain only 1 cm of length from the proximal nerve. This is not enough to bridge the gap in this scenario.

Any attempt at primary repair of a nerve injury with a 3-cm gap would require nerve grafting. In a mixed nerve, multiple cables of nerve graft are recommended in an attempt to topographically reconnect the sensory and motor fascicular bundles. The current gold standard for nerve repair in adults is autograft. The most common donor nerves are the sural nerve and medial antebrachial cutaneous nerve.

Processed nerve allograft has become a viable alternative to autograft nerve. The allograft nerve is processed and decellularized but maintains the microstructure of the nerve tissue including the fascicular anatomy and microvasculature. The allograft is rapidly revascularized without the donor site morbidity associated with autograft. The RANGER study has demonstrated S3 and M4 or above recovery in 86% of repairs using allograft nerve in gaps up to 5 cm. This question specifically asks about the most rapid recovery of motor function, which should occur with a nerve transfer.

2017

How well did you know this?
1
Not at all
2
3
4
5
Perfectly
14
Q

A 30-year-old man undergoes evaluation of a stab wound to the left upper extremity. A photograph is shown. Examination shows that the injured nerve is completely transected. Supination is weak, and he cannot flex his elbow in a supinated position. Which of the following areas is most likely to be insensate?

A) Dorsal little finger
B) Index finger pad
C) Posterior arm
D) Radial forearm
E) Thumb pad

A

The correct response is Option D.

The patient has an injury to the musculocutaneous nerve. This nerve provides motor axons to the brachialis, biceps brachii, and coracobrachialis. Patients with a musculocutaneous nerve transection cannot flex the elbow when supinated; the brachioradialis would provide some elbow flexion in a pronated position. The biceps brachii is the strongest supinator of the forearm, so patients with this injury have weak supination. The musculocutaneous nerve also provides sensory axons to the lateral brachial and lateral antebrachial cutaneous nerves. Patients with transection of this nerve would be insensate on the lateral arm and the radial side of the forearm.

Sensation to the posterior arm is provided by the posterior brachial cutaneous nerve, a branch of the radial nerve.

Sensation to the thumb pad is provided by the median nerve.

Sensation to the dorsal little finger proximally is provided by the dorsal sensory branch of the ulnar nerve and distally by the ulnar and radial proper digital nerves, branches of the ulnar nerve.

Sensation to the index finger pad is provided by the median nerve.

2017

How well did you know this?
1
Not at all
2
3
4
5
Perfectly
15
Q

A 25-year-old woman comes to the office because of nerve compression of the right upper extremity. Electromyography and nerve conduction studies are planned. Which of the following is the most likely indicator of motor axon loss in this patient?

A) Absent polyphasic waveforms
B) Decreased distal motor latency
C) Fibrillation potentials
D) Increased amplitude
E) Increased conduction velocity

A

The correct response is Option C.

Specific electrodiagnostic criteria indicate axonal loss: nerve conduction study amplitudes are decreased, conduction velocity is slowed, distal latency is prolonged, and fibrillation potentials and polyphasic waveforms are present.

2017

How well did you know this?
1
Not at all
2
3
4
5
Perfectly
16
Q

A 50-year-old man who is homeless is brought by ambulance to the emergency department. His blood alcohol concentration is 325 mg/dL. Examination of the right hand and forearm shows absent palpable pulses at the radial and ulnar arteries. Compartment pressure is 55 mmHg. Which of the following nerves is most likely irreversibly affected in this patient?

A) Lateral antebrachial
B) Medial antebrachial
C) Median
D) Radial
E) Axillary

A

The correct response is Option C.

The most appropriate answer is median. Pathophysiology of Volkmann’s contracture begins with the deep and central muscles, which include flexor digitorum profundus and flexor pollicis longus. The next affected is the middle layer, which includes flexor digitorum superficialis and pronator teres and then the wrist flexors. Lastly, the extensor forearm is affected.

In terms of nerve sensitivities, beginning at 30 mmHg, there are decreased conduction velocities. At 50 mmHg, there is no conduction. After 8 hours, there is irreversible damage.

The median nerve is affected before the ulnar nerve. The radial nerve is dorsal and not in the deep compartment. Both antebrachial nerves are superficial. The axillary nerve does not go to the forearm.

Alcohol is a clear comorbidity in this patient and therefore the timing is unknown. The pulselessness in this case indicates a late finding of compartment syndrome. Pain out of proportion along with paraesthesias and pressure is an early sign. Other late signs include pallor and paralysis.

2017

How well did you know this?
1
Not at all
2
3
4
5
Perfectly
17
Q

A 24-year-old man is brought to the emergency department 5 hours after being involved in a motorcycle collision. The patient reports worsening pain of the left forearm despite previous opioid administration. On physical examination, the left radial pulse is easily palpable. Hypoesthesia in a median nerve distribution is noted in the left hand. Active motion of wrist and fingers is present but is minimal and limited by pain. Passive wrist motion produces intense pain in the left forearm. X-ray study of the left upper extremity shows no fracture, and remaining trauma workup is negative. A photograph is shown. Which of the following is the most appropriate next step in management?

A) Carpal tunnel release
B) Forearm fasciotomy
C) Local wound care until demarcation of tissue loss
D) Splinting and observation
E) Wound debridement and skin grafting

A

The correct response is Option B.

This patient presents with acute compartment syndrome (ACS) in the left forearm after blunt soft tissue trauma. The most appropriate next step in management is emergent forearm fasciotomy for decompression.

The diagnosis of ACS in the extremities should be based on clinical presentation and progress. Signs and symptoms commonly evolve over a few hours after the initial traumatic event, especially after fluid resuscitation. Occasionally, the patient’s initial presentation may raise strong suspicion and warrant immediate decompression. ACS in the extremities may develop from soft tissue injury without a fracture in up to 30% of cases.

The commonly accepted clinical findings suggestive of ACS (rest pain, pain on passive motion, paresthesia, and paresis) have been shown to have high specificity but low sensitivity, rendering them a poor predictive value. Paresis/paralysis and other signs of arterial obstruction (pulselessness, pallor, pain out of proportion) are thought to be particularly late findings. Therefore, physicians should have a high level of suspicion for ACS in any case of limb trauma associated with excessive pain and paresthesia. Measurement of compartment pressures may be of diagnostic assistance in equivocal cases, especially when the patient is unable to communicate.

Carpal tunnel release is an important component of a forearm fasciotomy; however, it is not sufficient decompression alone. Emergent decompressive fasciotomy should precede wound debridement, demarcation of tissue loss, and splinting of the extremity. Skin grafting of the wounds (including the skin incisions for fasciotomy) may or may not be needed after resolution of limb edema.

2017

How well did you know this?
1
Not at all
2
3
4
5
Perfectly
18
Q

A 45-year-old man sustains a laceration of the ulnar nerve proximal to the elbow. He has loss of intrinsic hand function. At the time of surgical repair, there is a 1-cm gap in the nerve. Which of the following is the most appropriate management of this injury?

A) Interposition nerve conduit
B) Interposition nerve grafting
C) Primary repair only
D) Primary repair with anterior transposition
E) Primary repair with distal nerve transfer

A

The correct response is Option E.

The most appropriate management of this injury is to repair the ulnar nerve primarily and perform a distal nerve transfer as well.

The ulnar nerve is one of the two upper extremity nerves that supply motor input to the intrinsic muscles of the hand (the other being the median nerve). It supplies the interossei, hypothenar muscles, ulnar lumbricals, and the adductor pollicis muscles. These small muscles of the hand are vital for proper thumb and finger function. Because of their small size and delicate nature, they are very sensitive to denervation, and over a period of 9 to 12 months atrophy beyond repair. The aim of nerve repair is to reestablish nerve signals to the end-organ (i.e., muscle) prior to irreversible denervated muscle. The injury described is a good 18 inches or so away from the hand, and regeneration of the motor fibers to the intrinsic muscles would only occur after at a rate of an inch a month, by which time permanent atrophy would have already occurred. Primary repair alone can lead to suboptimal intrinsic muscle function despite excellent technical repairs. Distal nerve transfers help prevent the denervational atrophy by “babysitting” the muscles during the time it takes for the ulnar nerve to regenerate its motor fibers to the end-organ/muscles. The terminal branch of the anterior interosseous nerve (AIN) is most commonly used as the donor nerve. An end-side neurorrhaphy is performed to the ulnar motor fascicles in the distal forearm, a distance which results in reinnervation of the intrinsic muscles well before the 9- to 12-month mark.

The known topography of the ulnar nerve allows the surgeon to coapt the donor nerves to the appropriate motor recipient site of the ulnar nerve. An end-end coaptation of the terminal AIN to the motor fascicle of the ulnar nerve is also an option.

Nerve grafts or conduits are not required to repair the nerve injury in this clinical scenario. A 1-cm gap can usually be primarily repaired after dissecting the nerve and freeing it up proximally and distally. If further length is needed for tension-free repair, the ulnar nerve can be transposed anteriorly out of its natural position, giving another few centimeters of length.

2016

How well did you know this?
1
Not at all
2
3
4
5
Perfectly
19
Q

A 35-year-old woman comes for evaluation of a 6-month history of increasing numbness of the right long finger of the dominant hand. She delivered a healthy newborn 6 months ago. She reports that the numbness awakens her from sleep and resolves after she shakes her hand. Physical examination shows a two-point discrimination of 5 mm in all digits. Which of the following is the most likely abnormal electrodiagnostic finding in this patient?

A) Motor action potential amplitude of 25 ?V
B) Motor latency of 4 ms
C) Occasional fasciculations
D) Positive sharp waves
E) Sensory action potential latency of 4 ms

A

The correct response is Option E.

The patient has early carpal tunnel syndrome. She only exhibits sensory findings with numbness that resolves. Two-point discrimination is abnormal in late carpal tunnel syndrome when there is irreversible nerve damage. There are no motor symptoms at this point.

Electrodiagnostic studies are a two-part examination consisting of sensory action potentials (SAP), also referred to as nerve conduction studies (NCS) and electromyography (EMG). Findings in EMG latency for muscle abnormality are >4.0 ms. Normal muscles show occasional fasciculations with high fibrillations 5 weeks after denervation. Normal motor nerve shows no sharp waves and no fasciculations. Mild motor latencies on NCS/EMG are >4.0 ms. Motor latencies of >6.0 are considered severe. Because this is early carpal tunnel syndrome with no evidence of motor weakness, a motor latency of 4.0 is unlikely. Similarly, a motor action potential of 25 ?V is unlikely.

Findings in SAP distal latency for sensory abnormality are >3.5 ms and an amplitude <15 ?V (normal is 15-25 ?V). Therefore, sensory action potential latency of 4.0 ms is correct.

2016

How well did you know this?
1
Not at all
2
3
4
5
Perfectly
20
Q

A 42-year-old man with carpal tunnel syndrome is evaluated for symptoms that are progressively worsening despite conservative management. Surgical release using an open, short scar technique is planned. Which of the following is the most accurate statement when comparing this technique with endoscopic release?

A) Both techniques are equivalent in long-term symptom relief and recovery
B) Endoscopic release is far superior because of superior long-term symptom relief
C) Endoscopic release is only used for bilateral cases
D) Open, short scar technique requires regional block, whereas endoscopic release does not
E) Open technique has a higher association with recurrent median nerve injury

A

The correct response is Option A.

Open, short incision and endoscopic carpal tunnel release are equivalent in long-term symptom relief and recovery. Carpal tunnel syndrome is a condition caused by compression of the median nerve at the wrist. It is characterized by pain and numbness of the fingers within the median nerve distribution: the thumb, index, and long fingers, as well as the radial aspect of the ring fingers. With progressive compression, thenar atrophy can occur as well as weakness of thumb opposition. Conservative treatment includes splinting, avoidance of repetitive activities or positions that elicit symptoms, and occasionally steroid injection. With progression of symptoms, surgical release is indicated.

Open release provides transcutaneous access to the transverse carpal ligament. Traditionally, a long incision had been used extending from the proximal palm across the wrist and onto the proximal forearm. The incision has become progressively shorter, such that most surgeons employ a short scar confined to the proximal palm. Endoscopic release uses two small incisions for port access and provides transection of the transverse carpal ligament without division of the palmar aponeurosis.

Debate has existed regarding the superiority or inferiority of one technique over the others. Clearly, the endoscopic technique causes less pain and less alteration in early grip strength, when compared with the more classic, longer incision open techniques. However, when specifically comparing the open, limited scar technique to the endoscopic technique, studies have shown essentially the same outcome data regarding strength, return to work, symptom relief, and reoperation.

Virtually all studies have shown that open and endoscopic release have the same long-term symptom relief, measured at multiple points in time up to one year.

The open technique is often thought to be associated with a lower association with recurrent median nerve injury.

Bilaterality does not preclude open or endoscopic release.

Both techniques can be done during local or regional anesthesia.

2016

How well did you know this?
1
Not at all
2
3
4
5
Perfectly
21
Q

A 68-year-old woman is evaluated because of numbness and paresthesias of the right hand. The diagnosis of carpal tunnel syndrome is confirmed by electromyography and nerve conduction velocity studies. The patient is interested in a trial of nonsurgical management. Nighttime splint immobilization and corticosteroid injection therapy to the carpal tunnel are planned. Which of the following best describes the most likely long-term outcome of this management strategy?

A) Complete resolution of the patient’s symptoms
B) No effect on the patient’s symptoms
C) No initial effect on the patient’s symptoms followed by gradual improvement over 6 to 12 months
D) Short-term improvement of the patient’s symptoms followed by a recurrence in 6 to 12 months
E) Worsening of the patient’s sensory and motor symptoms

A

The correct response is Option D.

Nonsurgical management of carpal tunnel syndrome has been shown to be effective for symptomatic relief of carpal tunnel syndrome for up to 3 to 6 months, depending on the type of intervention that is chosen. Most patients who choose nonsurgical management will ultimately require surgery for carpal tunnel syndrome. Studies suggest that up to 70% of patients will go on to surgical carpal tunnel release at 1 year.

Various nonsurgical interventions have been recommended for carpal tunnel syndrome. These include splint immobilization, oral corticosteroid therapy, local corticosteroid injection, ultrasonography, acupuncture, nonsteroidal anti-inflammatory drug (NSAID) therapy, and more. Many of the interventions are not supported by high-level evidence.

Studies have shown improvement in carpal tunnel syndrome with splint immobilization, oral corticosteroid therapy, local corticosteroid injections, and ultrasonography when compared with a placebo or no treatment. Local corticosteroid injection into the carpal tunnel is more effective than oral corticosteroid therapy at 1 and 3 months without the potential risks of systemic steroid therapy. Splint immobilization plus a cortisone injection is more effective than splint immobilization only at 6 months. Full-time splint immobilization has not shown any benefit when compared with nighttime splint immobilization only.

In patients who have mild to moderate carpal tunnel syndrome without static numbness, weakness, or thenar atrophy, short-term improvement or resolution of symptoms can be expected but long-term resolution is not likely.

Patients with severe disease and signs of longstanding nerve compression including numbness and loss of abductor pollicis brevis strength may not respond to nonsurgical interventions. Worsening of symptoms as a result of nonsurgical treatment is unlikely.

Carpal tunnel release has been shown to be more effective than splint immobilization and corticosteroid injections for symptomatic relief at 3, 6, and 12 months. However, many surgeons will offer a trial of nonsurgical management based on patient preferences. Complete resolution of symptoms following nonsurgical intervention is thought to be a good prognostic indicator for the success of surgery.

2016

How well did you know this?
1
Not at all
2
3
4
5
Perfectly
22
Q

A 26-year-old man is referred for evaluation 9 months after sustaining a lower trunk brachial plexus traction injury after being hit by a motor vehicle. At the time of injury, he had paresthesia in the ring and small fingers and weakness of grip. Today, physical examination shows intrinsic atrophy and mild ulnar claw hand. A photograph is shown. He demonstrates increased sensation in the ring and small fingers since the time of the accident, but he has no clinical or electrodiagnostic improvement in motor function. Which of the following is most likely to improve the claw hand posture?

A) Anterior interosseous nerve branch of median nerve to ulnar nerve transfer in forearm
B) Brachialis branch of musculocutaneous nerve to ulnar nerve transfer in upper arm
C) Exploration and neurolysis of the lower trunk of the brachial plexus
D) Posterior interosseous nerve branch of radial nerve to ulnar nerve transfer in forearm

A

The correct response is Option A.

Lower trunk brachial plexus injuries can lead to deficits in hand function. Some can cause global hand dysfunction (median and ulnar nerve), while others can present as isolated ulnar nerve dysfunction. The signs and symptoms of isolated ulnar nerve dysfunction include numbness and tingling in the ring and small fingers, as well as weakness of the intrinsics. Ulnar nerve dysfunction may or may not present with clawing of the ulnar two digits, depending upon whether the flexor digitorum profundus (FDP) tendon to the ring and small fingers is involved. This patient has signs and symptoms consistent with low ulnar nerve palsy, and would benefit from an anterior interosseous nerve (AIN) to ulnar motor nerve transfer in the forearm.

The brachialis branch of the musculocutaneous nerve is classically used to reinnervate the AIN in the upper arm. This transfer is employed in cases of AIN palsy. The extensor carpi radialis brevis branch of the posterior interosseous nerve is most commonly used to reinnervate the AIN (median nerve), but can also be transferred to the ulnar nerve. This transfer occurs more proximally in the forearm and, therefore, would require a greater distance to travel to reach its endpoint (intrinsics of hand). The better and more appropriate transfer is the AIN to ulnar motor in this case.

Internal neurolysis can be used for neuromas in situ or neuropraxic nerves, but it would not be indicated for deficit in the hand. The length needed to recover function is too great, and motor end plate death is likely to occur before meaningful recovery can be seen.

2016

How well did you know this?
1
Not at all
2
3
4
5
Perfectly
23
Q

A 6-month-old boy who sustained a brachial plexus injury during delivery is brought for evaluation. On examination, the left elbow is held in extension and arm is positioned in internal rotation. He has active flexion of the fingers and wrist. Extension of the fingers and wrist is weak; anti-gravity test of the elbow extension discloses weakness. There is no active elbow flexion or shoulder abduction; Horner’s sign is absent. Which of the following structures is most likely injured?

A) C7 roots
B) C8-T1 roots
C) Lateral cord
D) Posterior cord
E) Upper trunk

A

The correct response is Option E.

The infant has the classic manifestations of an upper trunk (C5-6), or Erb’s, palsy (weak or absent elbow flexion, shoulder abduction and external rotation, relatively preserved elbow extension and distal wrist/hand flexion). The “waiter’s tip” posture of the affected extremity indicates relative sparing of lower root (C8-T1) function. Isolated injury to the C7 root is uncommon and would primarily affect radial nerve innervated muscles such as the wrist and finger extensors, and forearm pronation. Injury to the posterior cord also effects radial nerve innervated muscles and would not alter biceps or deltoid function as observed in this child. Lateral cord damage would result in weak or absent biceps and pectoral function, but would not diminish deltoid or periscapular muscle activity.

2016

How well did you know this?
1
Not at all
2
3
4
5
Perfectly
24
Q

A 14-year-old boy sustains a laceration of the distal forearm. Physical examination and wound exploration suggest flexor carpi ulnaris tendon and ulnar nerve involvement. The patient undergoes immediate microsurgical nerve repair with a nerve graft. Which of the following factors is most likely to predict a satisfactory outcome in this patient?

A) Age
B) Gender
C) Immediate repair
D) Ulnar nerve involvement
E) Use of nerve graft

A

The correct response is Option A.

Multiple studies have evaluated outcomes of median and ulnar nerve repair after transection injury. A meta-analysis by Ruijs et al. confirmed that younger age, specifically under 16 years old, was associated with the highest chance of satisfactory recovery of motor function in both median and ulnar nerve injuries. Patient gender was not found to be a statistically significant factor in outcome. In the same analysis, median motor nerve injuries were found to have a better chance of recovery than ulnar motor nerve injuries. Timing influenced outcome, with delay of repair adversely affecting prognosis, and although the ideal window for repair was not able to be defined by this review, there is some evidence that immediate repair may result in worse outcomes. The use of nerve grafts did not significantly predict motor recovery in these injuries.

2015

How well did you know this?
1
Not at all
2
3
4
5
Perfectly
25
Q

A 65-year-old woman undergoes open reduction and internal fixation for the injury displayed in the x-ray study shown. Six weeks postoperatively, she has diffuse swelling, stiffness, and pain of the right upper extremity. Her skin appears shiny, and she has changes in the patterns of both hair and perspiration in the right upper extremity when compared with the left upper extremity. After injury, treatment with which of the following would most likely have decreased her risk for this complication?

A) Amitriptyline
B) Ascorbic acid
C) Gabapentin
D) Prednisone
E) Pregabalin

A

The correct response is Option B.

Complex regional pain syndrome (CRPS) is chronic pain that persists in the absence of ongoing cellular damage and is characterized by autonomic dysfunction, trophic changes, and impaired function. In the perioperative period, the physiologic consequences of CRPS in the upper extremity contribute to or create one or more of the following: clinically significant osteopenia, delayed bony healing or nonunion, joint stiffness, tendon adhesions, arthrofibrosis, pseudo-Dupuytren palmar fibrosis, swelling, and atrophy. The reported incidence of CRPS is 5.5 to 26.2 per 100,000 person-years, and the prevalence is reported as 20.7 per 100,000 person-years. Women are more frequently affected than men, with a ratio of 3:1 to 4:1; the upper extremity is involved more frequently than the lower extremity; and fracture is the most common causative event. Incidence of CRPS after distal radius fracture has been reported to be between 22 and 39%.

In a double-blind, prospective, multicenter trial by Zollinger et al., 416 patients with 427 wrist fractures were studied for the effects of prophylactic vitamin C (ascorbic acid) on the risk of subsequent development of CRPS. Administration of 500 mg vitamin C daily was found to significantly decrease the incidence of CRPS in patients with distal radius fracture. The authors recommend treatment for 50 days.

The other four medications listed have all been reported for the treatment of patients diagnosed with CRPS. None have been reported to decrease the incidence of CRPS when used prophylactically.

2015

How well did you know this?
1
Not at all
2
3
4
5
Perfectly
26
Q

A 30-year-old man is evaluated after sustaining a laceration of the left ulnar nerve at the level of the mid humerus. Exploration and direct repair of the ulnar nerve are performed immediately. Which of the following interventions is most likely to result in optimal recovery of hand function?

A) Anterior interosseus nerve transfer to the ulnar nerve
B) Dynamic splinting
C) Electrical stimulation of the ulnar nerve
D) Flexor digitorum superficialis tendon transfer to the A2 pulley
E) Wrist extensor tendon transfer to the lumbricals

A

The correct response is Option A.

High ulnar nerve injuries are plagued by poor recovery of intrinsic function even when the nerves are repaired under optimal conditions. To maximize recovery and restore intrinsic function in the hand, one of the most effective interventions is transfer of the motor branch of the anterior interosseus nerve to the motor branch of the ulnar nerve at the wrist. Recently, this has been shown to also be useful as a “babysitter” transfer, potentially retaining motor endplates in the small muscles of the hand while a primarily repaired ulnar nerve regenerates.

Dynamic splinting has played a role in radial palsies and is frequently employed to overcome joint contractures that are not fixed (with a hard endpoint). This would not assist in hand recovery in this setting.

Electrical stimulation performed by a hand therapist helps in retraining innervated muscles but will not improve function in the denervated small muscles of the hand.

Tendon transfers have been the mainstay of functional restoration when nerve repair is not possible and/or the timeframe that has elapsed is great enough to make successful reinnervation of the target muscles unlikely. Both the wrist extensor-to-lumbrical transfer and the flexor digitorum superficialis transfer have been used to correct the “clawing” associated with ulnar palsies.

2015

How well did you know this?
1
Not at all
2
3
4
5
Perfectly
27
Q

A 45-year-old right-hand–dominant man who is a tennis player is evaluated because of a 6-month history of pain in the right lateral elbow. He has pain when lifting objects, and the pain radiates to the forearm. Physical examination shows tenderness just distal and anterior to the lateral epicondyle. Which of the following muscles is most likely affected?

A) Brachioradialis
B) Extensor carpi radialis brevis
C) Extensor carpi radialis longus
D) Extensor carpi ulnaris
E) Extensor digiti minimi

A

The correct response is Option B.

The extensor carpi radialis brevis (ECRB) origin is the primary muscle involved in lateral epicondylitis. The undersurface is avascular, making it a potential site for degeneration and partial tears. The ECRB shares a common origin with the extensor carpi ulnaris, extensor carpi radialis longus, and brachioradialis. The extensor digiti minimi also originates from the lateral epicondyle and has been involved in some cases of lateral epicondylitis, but not as commonly as the ECRB.

2015

How well did you know this?
1
Not at all
2
3
4
5
Perfectly
28
Q

Stellate ganglion blocks help alleviate the symptoms of complex regional pain syndrome in which of the following ways?

A) Helping repair nerve injuries
B) Increasing parasympathetic tone
C) Reducing sympathetic tone
D) Relaxing the muscles of the upper extremity
E) Releasing the body’s natural endorphins

A

The correct response is Option C.

Complex regional pain syndrome (CRPS) is a long-term pain condition that is believed to result from dysfunction in the central or peripheral nervous systems. CRPS is characterized by pain, swelling, or stiffness in the affected hand or extremity. The pain may be out of proportion to the injury that triggers it.

There are two types of CRPS. In type I, there is no identifiable nerve injury. In type II, there is an identifiable nerve injury. The name of this disorder was changed from reflex sympathetic dystrophy to CRPS because not all patients have increased sympathetic tone.

Stellate ganglion blocks may be used to anesthetize the stellate ganglion, which is a cluster of sympathetic nerves at the base of the neck, in an effort to decrease the overactivity of the sympathetic nerves seen in CRPS. The sympathetic nervous system has been implicated in the pathophysiology of CRPS, and consequently, sympathetic nervous system blockade is widely used to treat CRPS.

Sympathetic nervous system dysfunction is presumed to be an essential component of the syndrome, and sympathetic blockade has been recommended as early as possible to interrupt and reverse the process. The treatment effectively cuts the vicious cycle of pain, immobilization, and decreased joint motion.

Stellate ganglion blocks do not act to increase parasympathetic tone, relax muscles, release endorphins, or repair injured nerves.

2015

How well did you know this?
1
Not at all
2
3
4
5
Perfectly
29
Q

A 35-year-old woman who underwent surgical release of the first dorsal compartment 8 months ago is evaluated because of severe pain over the anatomical snuffbox of the right hand since the surgery. Physical examination shows focal tenderness over the region of the first dorsal compartment, a Tinel sign at the surgical scar, and numbness distal to the surgical scar. Medication includes gabapentin since the pain began, and the patient has attempted desensitization in hand therapy. Which of the following is the most appropriate next step in management?

A) Exploration silicone capping of the palmar cutaneous nerve
B) Initiation of oral pregabalin therapy
C) Iontophoresis with dexamethasone
D) Neurolysis and intramuscular placement of the superficial radial nerve
E) Phenol injection into the scar and compression glove therapy

A

The correct response is Option D.

One potential complication from surgical treatment of de Quervain disease (first dorsal compartment release) is injury to branches of the radial sensory nerve. When such injury is suspected, treatment should consist of exploration and, if possible, repair of the injured nerve branch. When this is not possible, multiple surgical options are available; transposition of the injured radial sensory nerve into the brachioradialis has been one of the commonly employed treatment strategies. Alternatives include transposition of the nerve into bone or a vein, nerve stripping, and coverage of the injured nerve with vascularized tissue.

Both gabapentin and pregabalin are useful medical interventions for nerve pain. General guidelines suggest proceeding with surgical treatment for suspected neuromas if no improvement occurs after 6 months of oral therapy.

Iontophoresis with corticosteroids has no proven efficacy for neuroma therapy. Phenol has been employed for stump neuromas and for Morton’s neuromas; however, there is no evidence to support its use in non-amputation neuroma treatment.

Silicone capping, once advocated for treatment of end-neuromas, has fallen out of favor because of poor pain relief results and displacement/migration of the caps.

2015

How well did you know this?
1
Not at all
2
3
4
5
Perfectly
30
Q

A 43-year-old woman is evaluated for intrinsic wasting and paresthesias of the little and ring fingers. Which of the following is the most likely site of nerve entrapment?

A) Anconeus epitrochlearis
B) Arcade of Frohse
C) Lacertus fibrosis
D) Ligament of Struthers
E) Pronator teres

A

The correct response is Option A.

Nerve entrapment is caused by the anconeous epitrochlearis. The scenario describes both motor and sensory signs and symptoms attributable to ulnar nerve compression. Ulnar nerve entrapment at the elbow is the second most common nerve entrapment neuropathy in the upper limb other than carpal tunnel syndrome in young adults. The most common cause is the ligament of Osborne. The anconeus epitrochlearis muscle, which is a congenital accessory muscle, arises from the medial epicondyle of the humerus and inserts at the olecranon process of the ulna. It can be found in normal elbows with an incidence of between 4 and 34%. It has been associated with ulnar nerve compression at the elbow.

The arcade of Frohse has been associated with posterior interosseous nerve entrapment. The lacertus fibrosis, pronator teres, and ligament of Struthers have been associated with median nerve compression. Note that the arcade of Struthers may be associated with ulnar nerve compression, but this option is not provided.

2015

How well did you know this?
1
Not at all
2
3
4
5
Perfectly
31
Q

A 36-year-old man comes to the office because of a 16-month history of diffuse pain of the posterior right shoulder. There is no history of trauma. Results of x-ray study are negative for an osseous injury. Physical examination shows atrophy isolated to the posterior scapular muscles. Motor and sensory examination of the right upper extremity shows weakness in shoulder external rotation with the arm adducted; no other abnormalities are noted. Which of the following nerves is most likely injured?

A) Axillary
B) Long thoracic
C) Spinal accessory
D) Suprascapular
E) Thoracodorsal

A

The correct response is Option D.

The patient has an isolated palsy of the suprascapular nerve, the first branch off of the upper truck (C5, C6) of the brachial plexus. Causes can include trauma, ganglion cyst (supraspinous fossa), or direct compression of the nerve as it passes under the transverse scapular ligament. Symptoms include diffuse posterior shoulder pain, atrophy of the supraspinatus and infraspinatus muscles, and weakness in shoulder external rotation.

The axillary nerve comes off of the posterior cord and innervates the deltoid muscle and teres minor muscle; palsy would primarily impair shoulder abduction. The long thoracic nerve is composed of contributions from the C5-C7 roots and innervates the serratus anterior muscle; a deficit would lead to scapular winging. The thoracodorsal nerve (C6-C8), a branch of the posterior cord, innervates the latissimus dorsi muscle; a deficit would impair shoulder extension, adduction, and internal rotation. The spinal accessory nerve innervates the trapezius and sternocleidomastoid muscles. Injury to this nerve does not affect the periscapular musculature.

2015

32
Q

A 44-year-old woman is evaluated because of a 6-month history of pain in her right upper chest and back, intermittent coolness in her right hand, and numbness and tingling of her right ring and little fingers. Results of the Adson test show a decreased radial pulse on the affected side, and the Roos test reproduces the patient’s symptoms on the affected side. Which of the following is the most appropriate next step?

A) Decompression of ulnar nerve at elbow
B) Noninvasive vascular study and electrodiagnostics
C) Resection of anterior and middle scalene muscles
D) Transaxillary resection of first rib
E) Observation

A

The correct response is Option B.

Three kinds of surgical procedures are employed to treat thoracic outlet compression syndrome (TOCS): transaxillary resection of the first rib, transcervical anterior and medial scalenectomies, and combined transaxillary first rib resection with immediate anterior and medial scalenectomies. This is the most complete procedure for total decompression of the thoracic outlet region. Because 70% of cases have soft-tissue involvement as the etiology of TOCS, current treatment includes transcervical anterior and middle scalenectomy in most TOCS cases.

Prior to any surgery, patients are treated conservatively with an exercise program for TOCS involving scalene stretching, first rib intercostal relaxation, nerve gliding, muscle relaxants, and pain patches for painful myofascial trigger points. Unfortunately, these conservative treatment modalities may yield only limited temporary help.

TOCS is usually classified in two groups. A neurogenic group comprises nearly 90% of all cases. This group usually has upper extremity pain, numbness, and tingling. A true vascular group comprises 10% of cases. Approximately 50% of patients still complain of coldness in the extremity. Approximately 40 to 50% of TOCS cases have concomitant peripheral nerve compression symptoms. Simple distal decompression of nerves will not usually lead to near-complete resolution of symptoms in cases of true TOCS.

There are two tissue groups that cause TOCS: soft tissue and osseous structures. The soft-tissue group includes anterior and middle scalene and their sheath, ligaments, and bands. This group comprises at least 70% of all TOCS cases because of congenital and acquired changes in the soft tissues. The osseous group comprises 30% or less of all TOCS cases and includes cervical rib, changes in the first rib, and clavicle due to injury.

Because TOCS can present with several different findings (including vascular and neurological compromise), it is advisable to work up these findings prior to committing to a treatment course. Noninvasive vascular studies and electrodiagnostics is the most reasonable first step in working up and treating these patients.

2015

33
Q

A 51-year-old woman is evaluated because of numbness and tingling of the dorsal and palmar aspects of the left hand, extending to the ring and little fingers, with worsening symptoms at night. Physical examination shows weakness of finger abduction in the hand. Which of the following is the most likely electrodiagnostic finding?

A) Decreased median conduction velocity from above elbow to wrist
B) Decreased ulnar conduction velocity from above elbow to wrist
C) Decreased ulnar conduction velocity from below elbow to wrist
D) Prolonged median sensory latency from wrist to digit
E) Prolonged ulnar sensory latency from wrist to digit

A

The correct response is Option B.

This patient exhibits symptoms of ulnar nerve compression at the cubital tunnel. Cubital tunnel syndrome is characterized by numbness and tingling in the ulnar nerve distribution (ulnar side of hand, involving little finger and ulnar half of ring finger) and can lead to intrinsic weakness. Compression occurs at the level of the elbow, with slowing of nerve conduction across the area of compression. Electrodiagnostic findings in ulnar nerve compression consist of decreased ulnar conduction velocity in the segment from above elbow to the wrist. Comparison of conduction velocities between above elbow to wrist and below elbow to wrist may show a difference, with nerve conduction being faster when measured from below the elbow, as the area of compression is not traversed. Conduction velocities from below elbow to wrist should not be affected. Ulnar nerve compression at the cubital tunnel can be distinguished from compression at the Guyon canal, as symptoms in the dorsal hand are not involved during nerve compression at the Guyon canal (at the level of the wrist), because the dorsal sensory branch of the ulnar nerve branches proximal to the wrist.

Prolonged median sensory latency from wrist to digit is seen in carpal tunnel syndrome.

2015

34
Q

A 32-year-old woman comes to the office for evaluation because of numbness of the left little finger 3 months after undergoing repair of a laceration of the left wrist sustained during an unsuccessful suicide attempt. Physical examination shows a healed laceration with a dysesthetic scar at the proximal wrist crease. A strong Tinel sign is present at the repair site. There is complete sensory loss of the little finger and no evidence of clawing. Motor function is intact. Wartenberg sign is absent. Two-point discrimination is greater than 15 mm. Which of the following is the most appropriate next step?

A) Microdissect the neuroma and identify motor fascicles with electrostimulation
B) Microdissect the neuroma and sural nerve graft fascicles
C) Resect the neuroma and direct repair with transposition
D) Resect the neuroma and repair with sural nerve grafts

A

The correct response is Option A.

The patient described has an ulnar neuroma-in-continuity with intact motor function and no sensory regeneration. Mackinnon has described an electrostimulation technique where the proximal motor fibers are identified using nerve stimulation.

Resection of the neuroma, with or without transposition, is not appropriate because it would cause damage to intact nerve fascicles. Microdissection without nerve stimulation would also cause damage to intact fascicles.

2014

35
Q

A 17-year-old boy is brought to the emergency department 5 hours after he sustained a stab wound to the left dorsal forearm. On physical examination, he is unable to extend the thumb and metacarpophalangeal joints. Exploration of the wound for repair of a presumed nerve injury is planned. Proper exposure of the nerve is between which of the following muscle groups?

A) Brachialis and triceps
B) Brachioradialis and extensor carpi radialis longus (ECRL)
C) Extensor carpi radialis brevis (ECRB) and ECRL
D) Extensor digitorum communis and ECRB
E) Flexor carpi radialis and pronator teres

A

The correct response is Option D.

Inability to extend the thumb and metacarpophalangeal joints generally suggests an injury to the radial nerve. Wrist extension can be preserved because of the preservation of the extensor carpi radialis longus muscle innervation. In the forearm, the radial nerve can be best approached directly between the extensor digitorum communis and the extensor carpi radialis brevis muscles. The approach between the brachialis and triceps muscles can identify the radial nerve in the upper arm. The interval between the flexor carpi radialis and pronator teres approaches the median nerve.

2014

36
Q

A 20-year-old man comes for evaluation 9 months after sustaining a stab wound to the left proximal upper arm. He did not seek medical attention at the time of the injury. Physical examination shows that he is unable to flex the left elbow with the forearm supinated. He is insensate to the lateral aspect of the upper arm and forearm. In addition to the ulnar nerve transfer to the biceps, which of the following nerve transfers is most appropriate to address this patient’s motor deficit?

A) Median nerve to brachialis
B) Musculocutaneous nerve to brachioradialis
C) Posterior interosseous nerve to triceps
D) Radial nerve to pronator teres
E) Ulnar nerve to flexor carpi radialis

A

The correct response is Option A.

The patient described has sustained a laceration of the musculocutaneous nerve. With this injury, the patient would be able to flex the elbow with the forearm in a pronated position using the brachioradialis, which is innervated by the radial nerve.

Due to the proximal level and amount of time that has passed since the injury, repair with grafting of the musculocutaneous nerve may not be advisable as the axons may not reach the motor end plates of the biceps and brachialis muscles before degeneration.

Use of fascicles from the median nerve, ulnar nerve, and both nerves has been described to restore elbow flexion. Fascicles are transferred distally in the upper arm directly to the nerve branch to the brachialis and/or biceps muscle. Distal coaptation allows donor axons to reach the target muscles more rapidly, and is more appropriate for this patient whose status is nearly 1 year post injury. A fascicle from the ulnar nerve was transferred to the nerve to the biceps (blue dot) and a fascicle from the median nerve was transferred to the nerve to the brachialis (green dot).

The musculocutaneous nerve is injured and cannot be used as a donor. The posterior interosseous nerve does not branch off the radial nerve until distal to the elbow; it would not be able to reach proximal enough to coapt to the nerve to the brachialis or biceps. The pronator teres does not flex the elbow; the median nerve is not injured. The flexor carpi radialis does not flex the elbow; the median nerve is not injured.

2014

37
Q

An 18-year-old man comes to the office because he has “drooping” of the right shoulder and inability to abduct it beyond 30 degrees. He underwent exploration and vascular repair 4 months ago after sustaining a stab wound to the right side of the neck. Photographs are shown. Which of the following surgical transfer techniques is most likely to restore deltoid function in this patient?

A) C5 to C6 nerve root
B) Intercostal to musculocutaneous nerve
C) Partial ulnar nerve to musculocutaneous nerve
D) Radial nerve fascicle to axillary nerve
E) Suprascapular to axillary nerve

A

The correct response is Option D.

Of the nerve transfers listed, only the transfer of a triceps branch to the axillary nerve will restore innervation to the deltoid muscle and provide shoulder abduction. Although debate remains over whether nerve graft repair (if possible) or nerve transfer provides better outcomes for shoulder abduction, both techniques are frequently employed in the setting of upper trunk brachial plexus injury.

The location of the injury (based on scar and mechanism) makes the injury likely distal to the nerve roots; in addition, root to root transfers are not typically part of the brachial plexus reconstruction ladder.

The suprascapular nerve is often the recipient nerve for partial transfers from cranial nerve XI (spinal accessory nerve) in order to provide shoulder stability. In this pattern of injury and based on the photos demonstrating atrophy of the supraspinatus and infraspinatus, the suprascapular nerve could not act as a donor nerve.

Intercostal nerve and partial ulnar nerve transfers to the musculocutaneous nerve are both frequently used to restore elbow flexion in the setting of upper trunk injuries.

2014

38
Q

In relation to the pronator teres muscle, which of the following is the most likely location of the median nerve in the proximal third of the forearm?

A) Between the superficial (humeral) and deep (ulnar) heads of the pronator teres muscle
B) Deep to the superficial (humeral) and deep (ulnar) heads of the pronator teres muscle
C) Superficial to the superficial (humeral) and deep (ulnar) heads of the pronator teres muscle
D) Through the deep (ulnar) head of the pronator teres muscle
E) Through the superficial (humeral) head of the pronator teres muscle

A

The correct response is Option A.

The course of the median nerve is relatively consistent. Just proximal to the elbow, at the medial epicondyle, there is a constant relationship of the median nerve, brachial artery, and the biceps tendon. From medial to lateral, the mnemonic, MAT, describes the relationship (Median nerve, brachial Artery, and biceps Tendon). In the cubital fossa, the nerve dives deep to the lacertus fibrosus, lying anterior to the brachialis muscle and medial to the brachial artery. The nerve enters the forearm between the superficial (humeral) and deep (ulnar) heads of the pronator teres muscle. As the nerve passes through the muscle bellies, it crosses the ulnar artery anteriorly, from medial to lateral, separated from the artery by the deep head of the pronator teres. After emerging from the pronator teres, the median nerve passes deep to an arch created by the two heads of the flexor digitorum superficialis. The nerve continues distally in the forearm between the flexor digitorum superficialis and flexor digitorum profundus. The nerve usually becomes superficial approximately 5 cm proximal to the wrist, emerging between the flexor digitorum superficialis and flexor carpi radialis, dorsal and slightly radial to the palmaris longus tendon.

2014

39
Q

A 24-year-old woman comes to the office because of severe thenar atrophy 3 years after failed repair of a low median nerve laceration. Tendon transfer to restore thumb opposition is planned. Optimal transfer would restore which of the following thumb functions?

A) Palmar abduction, extension, supination
B) Palmar abduction, flexion, pronation
C) Palmar abduction, flexion, supination
D) Radial abduction, extension, pronation
E) Radial abduction, flexion, supination

A

The correct response is Option B.

Injury to the median nerve, either by laceration or compression, results in thenar atrophy and loss of thumb opposition. Tendon transfer is the only reliable technique to restore thumb function in the face of severe, long-standing atrophy. Thumb opposition is a composite movement comprised of palmar abduction, flexion, and pronation. Opposition positions the thumb for grasp, but is not synonymous with it. There are numerous tendon transfer procedures described to improve thumb opposition (e.g., palmaris longus, abductor digiti minimi, flexor digitorum superficialis), but the most effective improve each of the three components. Thumb extension, supination, and radial abduction (in the plane of the hand) are not movements involved in opposition.

2014

40
Q

A 45-year-old man who is a cyclist comes to the office because of tingling of the left ring and small fingers. Normal sensibility dorsally is noted. Guyon canal release is planned. Which of the following structures is a border of the Guyon canal?

A) Capitate
B) Dorsal distal radio-ulnar ligament
C) Lunate
D) Pisohamate ligament
E) Volar distal radio-ulnar ligament

A

The correct response is Option D.

Guyon canal, also known as ulnar canal and ulnar tunnel, allows passage of the ulna nerve and artery into the hand. It is a semi-rigid fibrosseous longitudinal tunnel, approximately 4 cm in length, beginning at the proximal edge of the transverse carpal ligament and ending at the hypothenar aponeurotic arch. The roof comprises the volar carpal ligament. The medial wall is the pisiform and pisohamate ligament. The lateral wall is the hook of the hamate. The floor is the flexor retinaculum and hypothenar muscles.

The volar and dorsal distal radio-ulnar ligaments are the thicker portions of the triangular fibrocartilage complex (TFCC) and do not contribute to Guyon canal.

The capitate and lunate are part of the floor of the carpal tunnel and also do not contribute to Guyon canal.

2019

41
Q

A 52-year-old man presents for evaluation of a claw deformity of the right ring and small fingers. Medical history includes an unrepaired low ulnar nerve injury sustained 30 years ago. Which of the following is the most likely pathophysiology of this patient’s deformity?

A) Unbalanced abductor digit minimi muscle
B) Unbalanced median and ulnar innervated intrinsic muscles
C) Weak thenar muscles
D) Weak ulnar innervated extrinsic flexor muscles
E) Weak ulnar innervated intrinsic muscles

A

The correct response is Option E.

Clawing after ulnar nerve injury includes hyperextension of the metacarpophalangeal (MCP) joints and flexion of the interphalangeal (IP) joints. The pathophysiology includes paralysis of the interossei and third and fourth lumbricals. Unopposed long extensors cause the metacarpophalangeal joints to fall into extension while the long flexors pull the proximal interphalangeal joints into flexion. This posture is the classical ‘claw hand.’

2019

42
Q

Myoelectric prostheses offer which of the following advantages over body-powered prostheses?

A) Higher durability
B) Lower cost
C) Lower frequency of adjustment
D) More complex motions performed
E) Shorter training time

A

The correct response is Option D.

Body-powered prostheses have been shown to have advantages in durability, training time, frequency of adjustment, maintenance, and feedback; however, they could still benefit from improvements of control. Myoelectric prostheses have been shown to provide greater range of motion including more complex movements involving multiple joints moving at the same time. Currently, evidence is insufficient to conclude that either system provides a significant general advantage. Prosthetic selection should be based on a patient’s individual needs and include personal preferences, prosthetic experience, and functional needs.

2019

43
Q

A 57-year-old woman comes to the office because of burning pain and stiffness of the right hand 8 weeks after closed treatment of a distal radius fracture. The patient reports that she has had difficulty sleeping and continues to have discomfort despite taking narcotics. On physical examination, the hand is shiny, swollen, and warm, and finger range of motion is decreased. There is hypersensitivity to light touch. X-ray studies show good alignment of the fracture. Electrodiagnostic testing shows no abnormalities. Bone scan shows increased periarticular uptake. Which of the following is the most appropriate diagnosis?

A) Complex regional pain syndrome
B) Factitious disorder
C) Midpalmar space abscess
D) Opioid addiction
E) Pain catastrophizing

A

The correct response is Option A.

The most appropriate diagnosis is reflex sympathetic dystrophy, or complex regional pain syndrome (CRPS) type I. This patient exhibits symptoms consistent with CRPS, which is a form of severe neuropathic pain. The diagnosis of CRPS involves history, physical examination, and diagnostic testing. In addition to pain out of proportion, other features must be present. These can include changes in blood flow, altered temperature perceptions, sudomotor activity, edema, and pigmentation changes. Although no specific test is pathognomonic, triple-phase bone scans are helpful in adding credence to the diagnosis. First- and second-phase bone scans may show asymmetric flow and autonomic dysfunction, while the third phase demonstrates increased periarticular uptake in multiple joints of the affected extremity.

CRPS is divided into two types. Type I occurs without identifiable nerve involvement (also known as reflex sympathetic dystrophy), and Type II has identifiable nerve involvement (causalgia). It is more common in smokers and in women. Pain in CRPS can be either sympathetically mediated or sympathetically independent. This condition is characterized by persistent pain, cold intolerance, autonomic dysfunction, and trophic changes. Patients may show swelling, stiffness, difficulty sleeping, and persistent pain out of proportion to the normal postoperative/post-injury course that may be incompletely relieved by narcotics.

A variety of treatment modalities have been employed in addressing CRPS. These range from therapy modalities such as range of motion, stress loading, and desensitization to pharmacologic interventions with anticonvulsants or antidepressants. Stellate ganglion blocks or autonomic nerve blocks may be helpful in sympathetically mediated pain, and nerve stimulation (either transcutaneous or at the spinal cord level) can also be employed. Often multiple modalities are used concurrently and in sequence. Peripheral nerve decompression may be helpful in resolving symptoms related to CRPS type II.

Factitious disorder can occur when there is potential for secondary gain, but would not present with physiological symptoms.

Although opioid addiction can be a source of pain complaints in an attempt to acquire additional narcotics, the patient exhibits physiological changes that are unable to be mimicked.

Pain catastrophizing is a maladaptive behavioral response to pain that can be a risk factor for prolonged pain after trauma.

A midpalmar space abscess would be unlikely after a closed distal radius fracture. It would also not be likely to have trophic skin changes or changes in a bone scan as seen in this patient.

2019

44
Q

A 26-year-old man comes to the office 4 weeks after injuring his left shoulder while snowboarding. Physical examination shows limited abduction and forward flexion of the shoulder to 30 degrees. No additional abnormalities are noted. From which of the following areas of the brachial plexus does the affected nerve most likely arise?

A) Lateral cord
B) Lower trunk
C) Medial cord
D) Posterior cord

A

The correct response is Option D.

The axillary nerve (ventral rami of C5 and C6) arises from the posterior cord of the brachial plexus, giving off muscular branches to teres minor and deltoid. It also innervates the shoulder joint and the skin over the deltoid. Its close proximity to the inferior shoulder capsule as it courses on the anteroinferior border of the subscapularis and then through the quadrangular space, puts it at risk for injury.

The axillary nerve is most commonly injured during orthopedic surgeries such as shoulder arthroscopy, and open reduction and internal fixation (ORIF) of the proximal humerus, in which case, it is most commonly the result of closed traction injury. It can also be seen in the setting of an anterior glenohumeral joint dislocation or proximal humerus fracture, or as the result of a direct blow to the superior aspect of the shoulder. The majority of nerve injuries are temporary neurapraxias, which typically resolve within 6 to 12 months of injury; however, permanent nerve deficit can occur, requiring surgical intervention in the form of decompression, or reconstruction with nerve graft or nerve transfer from the radial nerve.

The lateral cord receives contribution from C5, C6, and C7 roots, and contributes to the musculocutaneous and median nerves. The medial cord receives contribution from C8 and T1 roots, and contributes to the median and ulnar nerves. There are no superior or anterior cords within the brachial plexus.

2019

45
Q

The Jones transfer for radial nerve palsy, specifically flexor carpi ulnaris to extensor digitorum communis III-V, is most likely to have which of the following significant disadvantages?

A) Difficulty of dissection
B) Inappropriate excursion of donor tendon
C) Indirect line of pull requiring pulley creation
D) Poor synergy
E) Unacceptable postoperative rupture rate

A

The correct response is Option B.

Although the Jones transfers were practiced for years, there were significant disadvantages that led to them falling out of favor. Among the disadvantages are loss of flexor carpi ulnaris (FCU) as an important ulnar wrist stabilizer and weakness in flexion/ulnar deviation, which is a very important wrist motion. Additionally, the short excursion of FCU is inadequate to fully extend the fingers when transferred to extensor digitorum communis (EDC).

Subsequent modifications to the Jones technique by Boyes and others found better alternatives to the use of FCU. Standard Boyes transfers are:

PT to ECRL and ECRB
FCR to EPL and ABL
FDS-III to EDC (via interosseous membrane)
FDS-IV to EPL and EIP (via interosseous membrane)

2019

46
Q

An 18-year-old man presents for follow-up evaluation 8 weeks after he sustained a penetrating injury to the posterior medial right elbow and a complete transection of the ulnar nerve in the cubital tunnel. At this time, which of the following are the most likely Sunderland/Mackinnon injury grade and electromyogram/nerve conduction findings in this patient?

A

The correct response is Option D.

Nerve injuries are graded using the Sunderland/Mackinnon classification. Grade I injuries involve neurapraxia and are expected to recover completely; grades II to IV injuries involve increasing disruption of the perineurium and endoneurium (with expectation for a variable degree of spontaneous recovery); grade V injuries represent neurotmesis, or complete transection of the nerve. Grade VI injuries represent combined injuries in which more than one grade of injury exists within the same segment of damaged nerve. After a complete nerve transection and progression of Wallerian degeneration, patients develop fibrillations and sharp waves and progressive decrease in the compound motor action potential.

2019

47
Q

A 32-year-old man presents to the emergency department for evaluation of a laceration of the right wrist sustained when he punched a glass window 1 hour ago. Physical examination shows a 2-cm transverse laceration of the volar ulnar wrist crease. Wound exploration shows complete laceration of the ulnar nerve. On physical examination of motor function, LOSS of which of the following functions is most likely in this patient?

A) Adduction of the thumb carpometacarpal joint
B) Extension of the metacarpophalangeal joint of the ring and small fingers
C) Extension of the thumb interphalangeal joint
D) Flexion of the interphalangeal joint of the index and middle fingers
E) Flexion of the interphalangeal joint of the ring and small fingers

A

The correct response is Option A.

The ulnar nerve is the terminal branch of the medial cord of the brachial plexus. It enters the forearm between the two heads of the flexor carpi ulnaris (FCU). In the forearm, the ulnar nerve innervates the FCU and flexor digitorum profundus of the small and ring fingers. It courses distally under the FCU to enter Guyon’s canal at the wrist. The dorsal cutaneous nerve, which gives sensation to the dorsoulnar hand, arises approximately 5 to 7 cm proximal to the ulnar styloid. In Guyon’s canal, the ulnar nerve splits into a deep motor and a superficial sensory branch. The deep motor branch innervates the hypothenar muscles (abductor digiti minimi, opponens digiti minimi, and flexor digiti minimi), as well as the lumbricals to the ring/small fingers, dorsal and palmar interossei, flexor pollicis brevis (deep head), palmaris brevis, and adductor pollicis. The superficial sensory branch in the palm innervates the small finger and the ulnar aspect of the ring finger.

Adduction of the thumb is controlled through activation of the ulnar-innervated adductor pollicis muscle. In the small and ring fingers, extension of the metacarpophalangeal (MCP) joint is performed through activation of the radially innervated extensor digitorum communis and extensor digiti minimi muscles. In the index, middle, ring, and small fingers, flexion of the proximal interphalangeal (PIP) joint is performed through activation of the median-innervated flexor digitorum superficialis.

While flexion of the ring and small finger distal interphalangeal (DIP) joints is produced by the ulnar-nerve innervated FDP tendons to the ring and small finger, the ulnar nerve provides branches to this muscle proximal to this patient’s injury. Extension of the thumb MCP joint is via the radial nerve innervated extensor pollicis brevis muscle.

2020

48
Q

In a transhumeral amputee, targeted muscle reinnervation can be utilized to improve control in a myoelectric prosthesis. Which of the following nerve transfers can be performed to provide intuitive prosthetic control for hand closure?

A) Median nerve to short head of biceps
B) Musculocutaneous nerve to long head of biceps
C) Radial nerve to lateral head of triceps
D) Radial nerve to long head of triceps
E) Ulnar nerve to lateral head of triceps

A

The correct response is Option A.

Targeted muscle reinnervation (TMR) utilizes a set of nerve transfers in order to allow intuitive prosthetic control for upper extremity amputees. Functioning nerves that no longer have their distal muscle target can be transferred to intact proximal muscles and generate a novel electrical signal that can be picked up by a myoelectric prosthesis. Another benefit of TMR is the potential to prevent or treat painful neuromas.

In the case of a transhumeral amputee, elbow flexion myoelectric prosthetic control is maintained by preserving musculocutaneous innervation to the long head of the biceps muscle. The distal remnant of the median nerve is transferred to the motor nerve of the biceps short head to create a signal for prosthesis hand closure. Elbow extension signals are maintained with radial innervation of the long head of the triceps. Signals for prosthesis hand opening are created with transfer of the distal radial nerve to the motor nerve of the triceps lateral head.

2020

49
Q

Resistance to which of the following maneuvers is most likely present in a digit that has intrinsic tightness?

A) Passive extension of the metacarpophalangeal (MCP) joint with the proximal interphalangeal (PIP) joint held in hyperextension
B) Passive extension of the PIP joint with the MCP joint held in hyperflexion
C) Passive flexion of the DIP joint with the PIP joint held in hyperextension
D) Passive flexion of the PIP joint with the MCP joint held in hyperextension
E) Passive flexion of the PIP joint with the MCP joint held in hyperflexion

A

The correct response is Option D.

The intrinsic muscles (dorsal/palmar interossei and lumbricals) are responsible for much of the fine motor function of the hand. Contractures of these muscles lead to a loss of the delicate and complex balance of the intrinsic and extrinsic muscles and typically results in the clinical picture of an intrinsic-plus hand. The intrinsics attach to the extensor mechanism through the lateral bands and facilitate force transmission from the muscles to the extensor mechanism on the proximal and distal phalanges. Because of their line of pull, the intrinsics are responsible for metacarpophalangeal (MCP) joint flexion and proximal interphalangeal (PIP) joint extension. The intrinsic tightness test (i.e. Bunnell test) requires one to assess passive PIP joint flexion with the MCP joint extended. This is compared with passive PIP joint flexion with the MCP joint in flexion which assesses for extrinsic tightness. If there is a substantial increase in resistance to PIP joint flexion with the MCP joint in extension, then the test is considered positive and indicative of intrinsic tightness or adhesions of the lateral bands.

Trauma is the most common cause of intrinsic muscle contracture. Spasticity from an upper motor neuron lesion (e.g. traumatic brain injury, cerebrovascular accident, cerebral palsy) may also lead to intrinsic contracture. Arthritis may also lead to intrinsic contracture resulting from joint deviation or dislocation.

In trauma, initial treatment is directed at edema prevention and aggressive hand therapy. Patients with spasticity from an upper motor neuron disorder are also initially managed with therapy and splinting. If these modalities are insufficient, surgical release of the intrinsic muscles or tendons (proximal or distal depending on extent of involvement) may improve posture and function. Ulnar motor neurectomy is another option in severely affected individuals to decrease intrinsic muscle tone and improve posture and function, but is only effective in the absence of a fixed MCP joint contracture.

2020

50
Q

A 37-year-old woman presents for evaluation of a laceration to the mid humerus that she sustained in a motor vehicle collision. On examination, the patient is unable to extend the wrist, fingers, or thumb. Surgical exploration shows complete radial nerve transection; the median/ulnar nerves are intact. Direct neurorrhaphy is performed after debridement and mobilization of the nerve ends. Which of the following is the last muscle to be reinnervated during nerve recovery?

A) Abductor pollicis brevis
B) Abductor pollicis longus
C) Brachioradialis
D) Extensor carpi radialis brevis
E) Extensor indicis proprius

A

The correct response is Option E.

Radial nerve injuries may occur in the setting of humeral fractures, and transection is most common in the setting of an open injury. The most important components of functional recovery following radial nerve injury include wrist, finger, and thumb extension. The order of reinnervation of the radial-innervated muscles is most commonly brachioradialis, extensor carpi radialis longus, supinator, extensor carpi radialis brevis, extensor digitorum communis, extensor carpi ulnaris, extensor digiti quinti, abductor pollicis longus, extensor pollicis longus, extensor pollicis brevis, and extensor indicis proprius. The abductor pollicis brevis is innervated by the median nerve via the thenar motor branch.

2020

51
Q

A 37-year-old woman presents with sharp lateral elbow pain sustained when lifting a garbage bag out of a can at work. Which of the following muscles is the most likely cause of this patient’s pain?

A) Anconeus
B) Brachioradialis
C) Extensor carpi radialis brevis
D) Extensor carpi radialis longus
E) Extensor carpi ulnaris

A

The correct response is Option C.

The common extensor tendon attaches the extensor carpi radialis brevis (ECRB), extensor digitorum communis (EDC), extensor digiti minimi, and extensor carpi ulnaris to the lateral epicondyle. Among these forearm extensor muscles, pathology found in the attachment of ECRB and EDC at the lateral epicondyle (LE) is commonly cited as a reason for pain at the LE. While the extensor carpi radialis longus (ECRL) is a wrist extensor and may be a source of lateral elbow pain with strain, it does not originate from the lateral epicondyle. The ECRL muscle was observed to originate from the distal aspect of the supracondylar ridge mainly as a muscular attachment. The brachioradialis is an elbow flexor and originates off of the lateral column of the distal humerus. It is not involved with lateral epicondylitis. The anconeus is a small muscle which originates off of the lateral epicondylitis and its main function is to assist with elbow extension.

2020

52
Q

A 36-year-old man presents with weakness of the left wrist and limited finger extension 2 days after he underwent surgery for a closed fracture of the right humerus that he sustained in a motor vehicle collision. At the time of surgery, the radial nerve was found to be intact without any significant signs of trauma. On examination, the patient is unable to extend his wrist or digits but elbow extension is intact. Which of the following histopathologic features seen in neurapraxia (Sunderland Type 1) injury is most likely in this patient?

A) Disruption of perineurium
B) Endoneurial fibrosis
C) Increased axonal transport
D) Segmental demyelination
E) Wallerian degeneration

A

The correct response is Option D.

Peripheral nerve injuries can be classified into three main categories: Neurapraxia, axonotmesis, and neurotmesis. These groups have been further classified by several different classification systems, which include those by Seddon and Sunderland, among others.

Neurapraxia (Sunderland type 1) is an injury to the myelin sheath only, while axons are preserved. In trauma, these injuries are most frequently caused by compression or stretching. Although segmental demyelination occurs (leading to conduction block), there is no Wallerian degeneration of the nerve, and a full recovery can be expected within days to weeks. Axonotmesis (Sunderland type 2, 3 or 4) involves damage to axons, and is characterized by Wallerian degeneration. Sunderland type 2 injury involves only the axons, and usually there will be a full recovery without intervention, while types 3 and 4 involve injury to the endoneurium and perineurium respectively and fail conservative management. Neurotmesis (Sunderland type 5) is a complete disruption of a peripheral nerve. MacKinnon and Dellon described a type 6 injury that involves mixed Sunderland type injuries along the length of a damaged nerve. Axonal transport is not a relevant histopathological feature.

2020

53
Q

A 50-year-old woman previously diagnosed with left forearm compression neuropathy of the superficial radial nerve comes to the office for examination. The patient has not responded to 7 months of conservative management which consisted of NSAID therapy, steroid injection, a trial of splinting, and activity modification. Operative treatment is planned. Fascial release between which of the following two tendons is most appropriate in this patient?

A) Abductor pollicis longus and extensor pollicis brevis
B) Brachioradialis and extensor carpi radialis longus
C) Extensor carpi radialis longus and extensor carpi radialis brevis
D) Flexor carpi radialis and abductor pollicis longus
E) Flexor carpi radialis and brachioradialis

A

The correct response is Option B.

Superficial radial nerve compression of the forearm occurs most frequently at the posterior border of the brachioradialis where the nerve transitions from a deeper, subfascial position to a more superficial, subcutaneous location. Also known as Wartenberg syndrome, patients may present with pain, numbness, or tingling over the dorsal radial hand radiating to the dorsal thumb and index finger.

Symptoms of superficial radial nerve compression may be confused with symptoms of de Quervain’s tenosynovitis. In addition, both conditions may coexist simultaneously.

Patients diagnosed with superficial radial nerve compression are initially treated conservatively since this approach is successful in relieving symptoms in the majority of cases. Conservative management consists of rest, splinting, removal of external compression source (such as a tight wristwatch band, bracelet, or handcuffs), and nonsteroidal anti-inflammatory medications.

Surgery is indicated when conservative measures fail. Surgical decompression involves release of the fascia between the brachioradialis and extensor carpi radialis longus tendons. It is at this interval that the nerve transitions from deep to superficial and prone to compression. The other responses do not reflect the correct surgical anatomy of this condition.

2020

54
Q

A 20-year-old woman presents for repair of a significant degloving injury to her dominant right thumb. During the soft-tissue repair, a 4-cm defect is noted in the radial proper digital nerve. Which of the following is the most appropriate technique for nerve repair?

A) Autologous nerve graft
B) Autologous vein graft
C) Collagen conduit
D) Nerve transposition

A

The correct response is Option A.

For nerve gaps 3 cm or longer that are not amenable to repair, peripheral nerve grafts are the most reliable choice. Proper microsurgical nerve repair requires meticulous, atraumatic technique with appropriate magnification, instrumentation, and sutures. Primary repair is recommended when a tension-free anastomosis is possible. In this case, primary repair is not possible, even with nerve mobilization or transposition. Use of these autologous grafts is limited by donor site supply and morbidity, loss of sensation, and possible neuroma formation.

Various nerve conduits such as silicone tubes, biologic tubes such as vein grafts, and biodegradable conduits such as collagen or polyglycolic acid are limited to small diameter nerves with shorter nerve gaps (2 to 3 cm). There appears to be no significant regeneration beyond 3 cm. Nerve transpositions are not performed for digital nerve repair.

2021

55
Q

A 29-year-old man is evaluated 15 months after a motorcycle collision in which he sustained a left humerus fracture and a complete left brachial plexus avulsion. He has not recovered any motor or sensory function of the left arm but desires the ability to actively flex the elbow. Which of the following procedures is most likely to restore active elbow flexion in this patient?

A) Free functioning gracilis muscle transfer
B) Pedicled bipolar latissimus dorsi muscle transfer
C) Proximal transfer of the flexor-pronator mass onto the humerus
D) Triceps-to-biceps transfer
E) Ulnar nerve to musculocutaneous nerve transfer

A

The correct response is Option A.

Adult traumatic brachial plexus injuries are devastating and life-altering injuries. Evaluation includes a detailed physical examination and radiologic and electrodiagnostic studies. Knowledge of injury patterns, timing of surgery, prioritization in restoration of function, and management of patient expectations are key components of management. In general, options for treatment of brachial plexus injuries include neurolysis, nerve grafting, or nerve transfers and should be performed within 6 months of injury. Free functioning muscle transfers (FFMT) and tendon transfers are typically used in patients who present late (greater than 12 months from injury), because the time for the nerve to regenerate after reconstruction is greater than the survival time of the motor end plates after denervation.

FFMT is the transplantation of a muscle and its neurovascular pedicle to a new location to assume a new function. The muscle is innervated by transferring an intact uninjured donor motor nerve; circulation is restored to the muscle through microsurgical anastomosis of the artery and vein to donor vessels (typically the thoracoacromial artery and cephalic vein). Within 6 to 9 months, the transferred muscle reinnervates, eventually gaining independent function. Although initially indicated in patients who presented late or as a salvage procedure with failed previous nerve reconstruction, the success with FFMT has led to use in early reconstruction to obtain elbow flexion and rudimentary grasp in patients with pan-plexus injuries. The gracilis muscle is the most commonly used donor because of its proximally based neurovascular pedicle and its long tendon length (which can reach distally into the forearm). Spinal accessory or intercostal nerves could be used as donor nerves for a free gracilis transfer.

Steindler flexorplasty, pedicled latissimus dorsi muscle transfer, triceps-to-biceps transfer, and Oberlin transfer each require remaining function of the brachial plexus and would be contraindicated in this patient with long-standing total brachial plexus palsy.

2021

56
Q

Which of the following diagnostic findings is most consistent with a recovering motor nerve injury?

A) Decreased motor unit potential amplitude
B) Decreased motor unit recruitment
C) Fibrillation potentials
D) Nascent potentials
E) Positive sharp waves

A

The correct response is Option D.

Electrical studies of a recovering nerve injury would show nascent potentials. This finding usually precedes the onset of clinically evident voluntary movement in the muscles innervated by the injured nerve. Nascent potentials appear several months after injury and result from axonal regeneration. Decreased motor unit potential amplitude, fibrillation potentials, positive sharp waves, and decreased motor unit recruitment are possible diagnostic findings in the setting of a nerve injury, but they are not indicative of recovery. Nerve lesions that spontaneously recover are usually treated nonsurgically, whereas those without recovery are explored and reconstructed. As a general rule, nerve regrowth occurs at approximately 1 inch per month or 1 millimeter per day. Motor endplates degrade at about 1% per week, hence the maximum length that a nerve can grow to restore motor function is approximately 13 to 18 inches. Repairs at the brachial plexus level rarely result in the recovery of any intrinsic muscle function. Sensory end organs, however, remain viable and can be reinnervated even after many years.

2021

57
Q

A male newborn is evaluated in the newborn nursery because of limited movement of the right arm. Maternal history includes gestational diabetes, but routine prenatal monitoring and ultrasound examinations were normal. The patient’s delivery was difficult, and he weighed 10.5 lb (4.8 kg) at birth. He was noted to have no spontaneous movement of the right arm. The right upper extremity was warm, pink, and supple. Pulsations of the radial and ulnar arteries were palpable at the wrist. X-ray studies of the affected shoulder show no obvious fractures. A photograph is shown. Which of the following is the most appropriate next step in management?

A) Angiography of the extremity
B) Anticoagulation
C) CT scanning of the extremity
D) Measurement of compartment pressures
E) Observation

A

The correct response is Option E.

This patient has an obstetrical palsy of the right upper extremity. The likelihood of recovery depends on the severity of the injury, but statistically over 70% of patients have complete or near complete recovery of upper extremity function without any surgical intervention. Thus the most appropriate next step for this newborn is observation. MRI of the shoulder and neck are helpful to discern evidence of anatomical injury to the cervical roots and/or portions of the brachial plexus, but CT scanning is unnecessary and of little use in this regard. Angiography and anticoagulation would be appropriate steps if there was clinical evidence of subclavian or brachial artery thrombosis, but the physical examination does not support this diagnosis. Similarly, the examination is inconsistent with neonatal compartment syndrome, a rare condition that usually presents with arm swelling, immobility, reduced arm perfusion, and purple cutaneous areas. Therefore, measurement of compartment pressures is unnecessary.

2021

58
Q

A 7-year-old boy falls and sustains the fracture in the x-ray study shown. He undergoes closed reduction and percutaneous pinning of the fracture that night. Postoperatively, he cannot flex his thumb at the interphalangeal joint or his index finger at the distal phalangeal joint. He has normal sensation of his thumb and other fingers. Which of the following nerves was most likely injured in the fall?

A) Anterior interosseous
B) Musculocutaneous
C) Posterior interosseous
D) Radial
E) Ulnar

A

The correct response is Option A.

The median nerve travels ulnarly to the brachial artery at the distal humerus. Because of the close proximity of the median and anterior interosseous nerves, they can be injured in pediatric supracondylar humerus fractures. This is often the result of strain on the brachial artery and median nerve at the time of injury, and nerve injuries occur in 11 to 16% of these fractures. Contusions of the median and anterior interosseous nerves often recover over time with conservative management. Patients can present with pure median nerve injury, pure anterior interosseous nerve injury, or injury to both nerves. In some cases of pulseless supracondylar humerus fractures, exploration is warranted. In this patient, motor deficits are indicative of an anterior interosseous nerve contusion.

Given the maintained sensation of the hand, the median nerve would be incorrect. The ulnar nerve does not provide motor innervation for thumb IP or index DIP flexion.The radial and posterior interosseous nerves do not innervate the flexor pollicis longus or index finger (flexor digitorum profundus) muscles and do not cross over the volar humerus. Also, the musculocutaneous nerve has no motor innervation distal to the elbow and is only sensory to the forearm.

2021

59
Q

A 55-year-old woman who is an administrative assistant is referred because of a 2-month history of numbness of the middle finger on the dominant hand. Splinting has not improved the symptoms. Electrodiagnostic studies show borderline mild carpal tunnel syndrome. Which of the following is the most appropriate next step in management?

A) Carpal tunnel release
B) Corticosteroid injection
C) Exercises with tendon and nerve gliding
D) Guyon canal release
E) Transcutaneous electrical nerve stimulation (TENS)

A

The correct response is Option B.

The answer is corticosteroid injection. A 3-month history of persistent symptoms is one indication for surgical intervention; the patient in this scenario has a history of 2 months. The patient has not had a trial of nonsurgical management. The electrodiagnostic studies show borderline or mild positive carpal tunnel syndrome. Corticosteroid injection has shown improvement in 32% of affected patients who did not have to go on to surgery. Corticosteroid injection should therefore be considered prior to surgical intervention. Transcutaneous electrical nerve stimulation as well as exercise has not been shown to be beneficial in the management of carpal tunnel syndrome. Besides corticosteroid injections, nonsurgical splinting by an occupational therapist and ultrasound have also been shown to improve symptoms of carpal tunnel syndrome.

2021

60
Q

A patient presents with an unresectable sarcoma of the proximal forearm. An elective transhumeral amputation is planned. Which of the following are the main advantages of performing targeted muscle reinnervation at the time of amputation in this patient?

A) Decreased pain in residual limb and improved control of myoelectric prosthesis
B) Greater length of residual limb and improved soft-tissue coverage over bone
C) Improved sensation in residual limb and greater range of motion in native elbow
D) Improved shape of residual limb and greater range of motion in native shoulder
E) Increased muscle bulk of residual limb and improved control of myoelectric prosthesis

A

The correct response is Option A.

Targeted muscle reinnervation (TMR) has been a major advance in the care of amputees and involves nerve transfers in the residual limb. Benefits of targeted muscle reinnervation include improved control of myoelectric prostheses for transhumeral amputees and an improvement in residual limb pain for amputees. There appears to be greater effect when TMR is performed acutely at the time of amputation. With this and other contemporary techniques for amputees, the nerves are provided a functional destination. TMR does not substantially alter the range of motion in the shoulder or elbow, and does not provide additional length of the limb. It does not appear to increase the muscle bulk in the residual limb.

2021

61
Q

A 65-year-old man presents to the office with symptoms and examination findings consistent with bilateral carpal tunnel syndrome. Medical history includes recent rupture of the left biceps tendon and several trigger fingers. Biopsy of the tenosynovium is performed during carpal tunnel release and the pathology report shows a positive Congo red stain. Which of the following diagnoses is most likely and should prompt further consultation?

A) Amyloidosis
B) Diabetes mellitus
C) Fibromyalgia
D) Malingering
E) Rheumatoid arthritis

A

The correct response is Option A.

A recent study showed that 10% of men over 50 years old and women over 60 years old with bilateral carpal tunnel syndrome had positive tenosynovial biopsies for amyloid. This can be a devastating disease if left to affect the heart or other organs and can be diagnosed with a simple biopsy during carpal tunnel release. Other findings suggestive of amyloidosis include a spontaneous biceps rupture, trigger finger, and spinal stenosis.

Patients with diabetes and rheumatoid disease have a higher incidence of carpal tunnel syndrome than the general population but the mention of biceps rupture points to the diagnosis of amyloidosis. Fibromyalgia and malingering have not been shown to be related to carpal tunnel syndrome.

2021

62
Q

A 15-year-old boy presents with a history of a traumatic laceration to the dominant right wrist after punching a window 4 weeks ago. He has numbness and weakness in the median nerve distribution. On operative exploration of the wound, there is a complete transection of the median nerve and a 4-cm gap between the proximal and distal ends. Which of the following is the best method of nerve reconstruction?

A) Polyglycolic acid nerve tube placement
B) Saphenous vein grafting
C) Sural nerve grafting
D) Transposition of the median nerve and primary nerve coaptation

A

The correct response is Option C.

The best method of nerve reconstruction is sural nerve grafting. An autologous sural nerve graft is the best choice for a nerve gap of 3 cm or greater. Multiple cables would likely be required to reconstruct the entire diameter of the median nerve and restore sensory and motor function.

Transposition of the median nerve and primary nerve coaptation is incorrect since the nerve gap is too large for a tension-free nerve repair, even with transposition of the median nerve at the wrist.

Saphenous vein grafting is incorrect since the size of the nerve defect warrants nerve grafting rather than vein grafting. Vein grafting could be used for an associated vascular injury of this size to the radial or ulnar artery.

Polyglycolic acid (PGA) nerve tube placement would be more appropriate for a nerve gap less than 3 cm. This serves as a scaffold for nerve regeneration. The indications for nerve allograft reconstruction are expanding the subject of ongoing research.

2022

63
Q

A 30-year-old man presents with an injury to the ulnar nerve at the elbow. Examination shows an 8-cm nerve gap. Reconstruction with distal nerve transfers at the level of the wrist (motor) and palm (sensory) is planned to restore intrinsic function and finger sensation. If the proximal nerve gap is not repaired, which of the following deficits will most likely persist?

A) Decreased elbow flexion strength
B) Decreased finger extension strength
C) Decreased sensation in the first web space
D) Decreased sensation on the dorsal hand

A

The correct response is Option D.

Distal to the elbow, the ulnar nerve supplies motor innervation to the flexor carpi ulnaris as well as the flexor digitorum profundus to the ring and small fingers, before heading toward the hand, where it supplies most of the intrinsic muscles of the hand (hypothenar muscles, ulnar two lumbricals, interossei, adductor pollicis, deep head of the flexor pollicis brevis). Along the way, it sends off a sensory branch to the dorsal hand; this branch arises about 7 cm proximal to the wrist crease. The most common nerve transfer for motor restoration is transfer of the nerve to pronator quadratus (distal anterior interosseous nerve) to the ulnar motor branch at the wrist. Sensory nerve transfers within the palm can restore sensation to the ulnar digits. However, these distal reconstructions will not restore sensation to the dorsal ulnar hand.

2022

64
Q

An upper trunk (C5-C6) brachial plexus injury is most likely to result in dysfunction of which of the following muscles?

A) Biceps
B) Flexor carpi ulnaris
C) Pectoralis minor
D) Trapezius
E) Triceps

A

The correct response is Option C.

Upper trunk brachial plexus injuries are associated with the classic “waiter’s tip” posture: the shoulder is adducted and internally rotated at rest. The elbow is extended, the forearm is pronated, and the wrist and digits are held in flexion. Impacted nerves include the axillary, musculocutaneous, and suprascapular nerves. The musculocutaneous nerve innervates the biceps and brachialis muscle. Loss of function of these muscles results in deficits of elbow flexion and forearm supination.

The flexor carpi ulnaris is innervated by the ulnar nerve, which originates from the medial cord of the brachial plexus and carries innervation from C8-T1.

The pectoralis minor is innervated by the medial pectoral nerve from the medial cord of the plexus and carries nerve fibers from C8 and T1.

The trapezius is innervated by the spinal accessory nerve (cranial nerve XI).

The triceps is innervated by the radial nerve, which originates from the posterior cord of the brachial plexus and has contributions from C5-T1.

2022

65
Q

Targeted muscle reinnervation allows for increased type of which of the following prosthetic controls for an above-elbow amputee?

A) Cable
B) Electromyographic
C) Passive
D) Switch

A

The correct response is Option B.

Upper limb amputation is a devastating loss, more so than lower limb amputation. The level of amputation portends increased functional loss, i.e., forearm-level amputees maintain more functional capacity than above-elbow amputees. Many patients with upper extremity amputations rely on their prosthetic for daily functional demands. Patients with an above-elbow amputation have less muscles to exert control of a prosthetic, which makes multiple simple movements difficult and compound movements especially challenging.

There are multiple types of prostheses for the upper extremity. There are passive prosthetics, which are frequently cosmetic but may allow for a stable post for the other extremity. Functional prosthetics are body-powered (via cable controls) or externally powered (myoelectric, switch control). Body-powered prostheses use the remnant body motions through a harness (i.e., scapular and humeral motion for a transhumeral amputee) via cable to control motion and the force of a terminal device. These are simple devices that do not require as much maintenance and have more longevity, but they are limited in terms of functionality.

Myoelectric prostheses use the remnant muscle contractions via electromyographic (EMG) capture for prosthetic control. The more independent discrete electromyographic signals there are, the more potential functional options of the prosthetic there are. If there are minimal/limited EMG sensor options, then switch control can be used. Switch control is when small switches are turned on/off to increase control for separate components of a prosthetic. Switch control can be combined with myoelectric control to increase functionality for higher level amputees. For example, a transhumeral-level amputee can use switch control to position the elbow, while EMG signals from the biceps and triceps control wrist/hand motion.

Targeted muscle reinnervation (TMR) uses nerve transfers to “hyperreinnervate” muscles to allow for increased distinct electromyographic signals as well as increased amplification of signals, increasing control sites for prosthetic functionality. A good example is using residual peripheral nerves of transhumeral amputees, such as the median, radial, and ulnar nerves, to increase independent control of each biceps and triceps head as well as brachialis. TMR therefore allows for increased EMG control of a myoelectric prosthetic.

2022

66
Q

A 35-year-old, right-hand–dominant man presents with a 4-month history of right upper extremity weakness. The patient reports a history of sudden onset shoulder pain that began without trauma. The pain improved, but he has developed difficulty abducting the shoulder, externally rotating the shoulder, and flexing the elbow. MRI shows hourglass constrictions of the brachial plexus. Electrodiagnostic studies demonstrate the involvement of multiple peripheral nerves. Which of the following is the most likely diagnosis?

A) Brachial plexus avulsion

B) Guillain-Barré syndrome

C) Parsonage-Turner syndrome

D) Rotator cuff injury

E) Transverse myelitis

A

The correct response is Option C.

Acute brachial neuritis, also known as neuralgic amyotrophy or Parsonage-Turner syndrome, is an uncommon but well-recognized clinical entity. Patients characteristically present with acute-onset unilateral severe shoulder and/or arm pain that radiates, followed by progressive weakness and atrophy of the muscles in the shoulder girdle and arm. Pain is the initial symptom for 90% of patients. The etiology of the syndrome is thought to be idiopathic; however, it has been reported in various clinical situations, including post surgery, post infection, during pregnancy, during vigorous exercise, post trauma, and post vaccination. Patients exhibit significant variability in presentation regarding nerve involvement, muscle involvement, the extent of recovery, and recurrence. Diagnosis is made by clinical examination and supported by electrodiagnostic studies and imaging. Electrodiagnostic studies show multiple peripheral nerve lesions, rather than a brachial plexus localization. Parsonage-Turner syndrome is considered self-limited and is often managed conservatively; however, recovery may be protracted and incomplete, and a large percentage of patients never fully recover (30%). Surgical exploration has become more common for patients who do not show clinical or electrodiagnostic signs of recovery after several months. Nerve imaging will demonstrate hourglass constrictions, which are neurolysed at the time of exploration, sometimes in conjunction with nerve transfer.

These patients usually have full shoulder range of motion; additionally, imaging does not correlate with rotator cuff injury.

Avulsion of the brachial plexus would have a prodrome of trauma, and diagnosis would point to root involvement as opposed to more distal nerve pathology.

Guillain-Barré syndrome is usually preceded by infection or other immune stimulation that induces an aberrant autoimmune response targeting peripheral nerves and their spinal roots. These patients traditionally present with symmetric acute flaccid paralysis that can progress to respiratory failure.

Transverse myelitis describes a heterogeneous group of inflammatory disorders that are characterized by acute or subacute motor, sensory, and autonomic (bladder, bowel, and sexual) spinal cord dysfunction (traditionally bilateral), which does not correlate with this patient’s presentation.

2022

67
Q

A Froment sign is created by retained function of which of the following muscles?

A) Adductor pollicis
B) Extensor pollicis brevis
C) Extensor pollicis longus
D) Flexor pollicis brevis
E) Flexor pollicis longus

A

The correct response is Option E.

Ulnar nerve injuries are particularly devastating and have significant sensory and motor functional consequences. Proximal to the elbow, the expected motor deficits include absence of the flexor carpi ulnaris, the flexor digitorum profundus to the middle/ring/small fingers, the hypothenar muscles (opponens digiti minimi, abductor digiti minimi, flexor digiti minimi brevis), the third and fourth lumbrical muscles, the dorsal and palmar interosseous muscles, the adductor pollicis, and the deep head of the flexor pollicis brevis. A Froment sign is observed when the patient attempts to pinch but is unable to activate the adductor pollicis (ulnar innervated) and compensates by activating the flexor pollicis longus by flexing the interphalangeal joint (median nerve innervated). Extensor pollicis longus (EPL), extensor pollicis brevis (EPB), and flexor pollicis brevis (FPB) are not involved in the thumb interphalangeal joint flexion that produces Froment sign.

2022

68
Q

Which of the following structures is a potential site for compression of the ulnar nerve?

A) Anconeus epitrochlearis
B) Lacertus fibrosis
C) Ligament of Struthers
D) Pronator teres
E) Supinator

A

The correct response is Option A.

Ulnar nerve compression at the elbow is the second most common compression neuropathy of the upper extremity (after carpal tunnel syndrome). There are several sites in this region that can contribute to ulnar nerve compression, including the arcade of Struthers, medial intermuscular septum, Osborne ligament, fascia between the two heads of the flexor carpi ulnaris (FCU), and fascial bands within the FCU distally. The anconeus epitrochlearis is an anomalous accessory muscle that can cause compression of the underlying ulnar nerve at the elbow. The anconeus epitrochlearis, present in 4 to 34% of individuals, originates from the medial epicondyle of the humerus and inserts into the olecranon. If identified during cubital tunnel surgery, this structure is typically released or excised.

The proximal leading edge of the supinator (arcade of Fröhse) can compress the posterior interosseous nerve.

The pronator teres muscle is a potential compression site of the median nerve in the proximal forearm.

The lacertus fibrosis (bicipital aponeurosis) is a potential compression site of the median nerve at the antecubital region.

The superficial radial nerve can be compressed between the tendons of the brachioradialis and extensor carpi radialis longus in the forearm.

The ligament of Struthers is a fibrous band that runs from the tip of the supracondylar process to the medial epicondyle. The median nerve and brachial artery can run through the passage created by the ligament and can be compressed when running through this passage. The arcade of Struthers differs from the ligament of Struthers in that the arcade is a fascial band that runs from the triceps fascia to the medial intermuscular septum, and it is a potential site of compression of the ulnar nerve.

2022

69
Q

A 29-year-old man who is a laborer presents with significant pain of the nondominant left hand. Medical history includes a laceration of the ring and small fingers palmarly over the proximal phalanx 1 year ago. At the time of the injury, he underwent repair of the flexor digitorum profundus to the small finger and direct repair of the ulnar digital nerve to the ring finger. All other nerves were intact. On examination, the patient reports pain over the nerve repair site that interferes with work-related tasks. There is no sensation distal to the repair. Positive Tinel sign over the ulnar extent of the scar on the ring finger is noted. Which of the following surgical techniques is most appropriate in this patient?

A) Excision of neuroma alone
B) Excision of neuroma and nerve implantation into bone
C) Excision of neuroma and nerve repair with allograft
D) External and internal neurolysis of the neuroma in continuity
E) Targeted muscle reinnervation or nerve transfer to nearby motor nerve

A

The correct response is Option C.

Injuries to the extremities are extremely common, and with the close proximity of the digital nerve to the palmar surface of the digits, these nerves are commonly lacerated. Initial treatment includes primary repair or grafting, depending on the extent of injury and the ability to perform direct repair with minimal tension. Despite repair, patients may develop neuroma in continuity and fail to recover meaningful sensation distal to the level of injury.

In this scenario, the patient has developed a neuroma in continuity at the site of digital nerve repair. Many treatment options exist for the treatment of neuroma in the extremities where the distal nerve target is present. These include: direct repair, the use of a nerve conduit, allograft, or autograft. Where the distal nerve target is absent, as with amputation, techniques to limit or treat neuroma include: excision alone, excision with amputation into muscle or bone, nerve graft to nowhere, nerve capping, centro-central coaptation, end to side neurorrhaphy, targeted muscle reinnervation (nerve transfer), or regenerative peripheral nerve interface (RPNI).

In the study by Moran et al., where authors retrospectively reviewed 127 patients following surgery for symptomatic neuroma, they found that patients who underwent neuroma excision followed by repair had significantly lower DASH scores, compared with implantation into muscle or bone or with simple excision alone (p = 0.03).

Lans et al. demonstrated that patients who underwent neuroma excision followed by repair/reconstruction had lower numeric rating pain scores, lower PROMIS pain interference scores, and higher PROMIS upper extremity scores.

While targeted muscle reinnvervation is a useful technique to prevent or treat neuroma pain, it is more commonly performed in the setting of amputation.

2022

70
Q

A 40-year-old man comes to the clinic because of difficulty with the use of his right hand. Six weeks ago, he was treated at the emergency department after sustaining multiple stab wounds to the right upper extremity. At that time, the wounds were irrigated and closed. Physical examination shows clawing of the small and ring fingers and hyperextension of the metacarpophalangeal joint of the thumb. Sensation is decreased over the palm but normal over the dorsum of the hand. Injury to which of the following nerves is the most likely explanation for these findings?

A) High median
B) High ulnar
C) Low median
D) Low ulnar
E) Radial

A

The correct response is Option D.

In low ulnar nerve injuries, the nerve is damaged distal to the motor branch of the flexor carpi ulnaris and motor branch to the flexor digitorum profundus (FDP) of the ring and little fingers. In low injuries, sensation over the palmar ulnar aspect of hand is lost and paralysis occurs, usually to all interosseous muscles, the two ulnar lumbrical muscles, the three hypothenar muscles, the adductor pollicis muscle, and the deep head of the flexor pollicis brevis muscle. Sensation over the dorsum of the hand may be intact if the lesion has occurred distal to the takeoff of the dorsal ulnar sensory nerve, which takes off approximately 5 to 6 cm proximal to the ulnar styloid. The loss of intrinsic muscle function results in an inability to flex at the metacarpophalangeal (MCP) joints and extend the interphalangeal (IP) joints. This results in the development of the intrinsic-minus or claw posture of the ring and little fingers, where there is hyperextension at the MCP joints and flexion at the IP joints (Duchenne sign). The development of clawing does require the presence of intact extrinsic extensor and flexor tendon function; therefore, a high ulnar nerve injury results in less claw hand deformity because of the loss of FDP function.

In addition, intrinsic weakness leads to decreased pinch and grip strength, a result of paralysis of the adductor pollicis, deep head of the flexor pollicis brevis, and first dorsal interosseus muscles. Thumb pulp to index pulp pinch leads to excessive flexor pollicis longus contraction and development of the Froment paper sign.

Median nerve sensation is intact, as is the function of the median innervated forearm and hand muscles. A high or low median nerve injury would not explain this patient’s symptoms. In low median nerve injuries, patients present with thenar wasting, weak thumb abduction and opposition, and decreased sensation of the volar thumb, index, and middle fingers. Patients with a high median nerve injury will have these same findings, as well as loss of FDP and flexor digitorum superficialis function to the index and middle fingers and loss of flexor pollicis longus function. They will also have weakness of the flexor carpi radialis muscle.

Distal to the elbow, the radial nerve divides into a deep branch and a superficial branch. The deep branch supplies motor function to the extensor carpi radialis brevis and supinator muscles. The deep motor branch continues as the posterior interosseous nerve, which supplies motor function to the abductor pollicis longus, extensor carpi ulnaris, extensor digiti minimi, extensor digitorum communis, extensor indicis proprius, extensor pollicis brevis, and extensor pollicis longus muscles. The superficial branch of the radial nerve supplies sensation to the dorsal radial hand, as well as dorsal sensation to the thumb, index finger, middle finger, and the radial side of the ring finger. Depending on the location and specific branches involved, radial nerve injury can result in loss of wrist, thumb, and finger extension, as well as dorsal radial hand and finger sensation.

2023

71
Q

A 54-year-old man is evaluated after sustaining a traumatic ulnar nerve laceration with no intrinsic muscle function. On examination, a Froment sign is noted. Which of the following muscles most likely compensates for the thumb adduction that caused the deformity?

A) Abductor pollicis brevis
B) Abductor pollicis longus
C) Extensor pollicis longus
D) Flexor pollicis brevis
E) Flexor pollicis longus

A

The correct response is Option E.

Ulnar nerve injuries have significant sensory and motor functional consequences. When injuries occur proximal to the elbow, the expected motor deficits are in the following muscles: flexor carpi ulnaris, flexor digitorum profundus to the ring and small fingers, hypothenar (opponens digiti minimi, abductor digiti minimi, flexor digiti minimi brevis), third and fourth lumbrical, dorsal and palmar interosseous, adductor pollicis, and deep head of the flexor pollicis brevis.

A Froment sign is observed when the patient attempts to pinch between the thumb and index finger but is unable to activate the adductor pollicis muscle (ulnar innervated) and compensates by activating the flexor pollicis longus muscle by flexing the interphalangeal joint (median nerve innervated). None of the other muscles listed cause adduction of the thumb to help with pinch between the thumb and index finger and therefore do not contribute to the deformity.

2023

72
Q

A 42-year-old man who sustained an amputation immediately distal to the elbow is scheduled to undergo a targeted muscle reinnervation procedure for improved prosthetic control and function. Which of the following nerve transfers is most likely to provide a “hand-close” signal in this patient?

A) Medial antebrachial cutaneous nerve to the brachioradialis muscle
B) Median nerve to the short head of the biceps
C) Musculocutaneous nerve to the pectoralis major muscle
D) Radial nerve to the lateral head of the triceps
E) Ulnar nerve to the brachialis muscle

A

The correct response is Option B.

Targeted muscle reinnervation techniques allow options for improved prosthetic function and nerve pain management in patients after traumatic amputations. The options available are determined by the available donor nerve remnants and muscle targets. A patient with a transhumeral amputation has an intact elbow joint but unusable forearm muscle targets. In such a patient, the intact elbow allows for maintenance of flexion and extension by preservation of musculocutaneous innervation of the long head of the biceps and radial innervation of the long head of the triceps. To provide “hand-close” prosthetic function, transfer of the median nerve remnant to the motor nerve of the short head of the biceps can be accomplished.

Transfer of the radial nerve remnant to the motor nerve of the lateral head of the triceps allows for prosthetic “hand-open” control. If available, an ulnar nerve remnant can be transferred to the nerve of the brachialis to provide prosthetic wrist control. Musculocutaneous nerve remnant transfer to the nerve of the pectoralis major is an option for patients with “shoulder disarticulation” amputations. Transfer of the medial antebrachial cutaneous nerve to the brachioradialis is an option for a patient with residual limb pain following a distal forearm amputation.

2023

73
Q

A 24-year-old man comes to the office after sustaining a flail upper extremity injury in a motorcycle collision 4 weeks ago. A severe brachial plexus injury is suspected. Which of the following evaluations will provide the most information to plan the best reconstruction for this patient?

A) CT myelography
B) Electromyography
C) MRI
D) Nerve conduction studies
E) Ultrasonography

A

The correct response is Option C.

Current MRI techniques have the ability to pick up root avulsions as well as other areas of nerve discontinuity. CT myelography has been used to determine the presence or absence of nerve root avulsion, but is not useful in identifying other nerve injuries. Ultrasonography can be useful to discern peripheral nerve anatomy but is not as sensitive as MRI. Nerve conduction studies and electromyography would not be useful until at least 6 weeks after injury.

2023

74
Q

A 22-year-old man comes to the emergency department with a sharp injury to the lateral cord of the brachial plexus sustained during an altercation. Which of the following examination findings is most likely present in this patient?

A) Intrinsic atrophy
B) Limited wrist extension
C) Loss of shoulder abduction
D) Numbness of the ulnar digits
E) Weak elbow flexion

A

The correct response is Option E.

The lateral cord of the brachial plexus contributes to the musculocutaneous nerve with its continuation of the lateral antebrachial cutaneous nerve, and it provides the sensory portion of the median nerve. Therefore, injury would affect elbow flexion as well as lateral forearm and median sensory innervation.

Loss of shoulder abduction (upper trunk, posterior cord), limited wrist extension (extended upper trunk, posterior cord), and ulnar numbness and intrinsic atrophy (lower trunk, medial cord) would be seen in other patterns of brachial plexus injury.

2023

75
Q

A 48-year-old man presents with an isolated nerve injury 4 weeks after sustaining a stab wound to the axillary region. On physical examination, he is unable to cross the index and middle fingers over one another. Surgical exploration with nerve repair is planned. Which of the following nerve transfers is most appropriate for this patient?

A) Anterior interosseous to ulnar
B) Median to the posterior interosseous
C) Musculocutaneous to anterior interosseous
D) Radial to axillary
E) Spinal accessory to suprascapular

A

The correct response is Option A.

This patient has sustained a high ulnar nerve injury proximal to the elbow. The primary deficits for this patient include ulnar hand sensation, as well as intrinsic hand dysfunction. In an adult, it is very challenging to restore intrinsic motor function in the setting of a high ulnar nerve injury. Options include ulnar nerve repair or reconstruction alone, although this strategy is unlikely to restore native intrinsic hand function. In this case, the best option is an anterior interosseous nerve to ulnar motor nerve transfer, which has been described and popularized during the past decades for high ulnar nerve injuries and in patients with a minimal chance of intrinsic recovery. In the scenario presented, a high ulnar nerve injury in an adult, the injury is unlikely to recover using intrinsic reconstruction with ulnar nerve repair or reconstruction alone, and a nerve transfer is the recommended strategy for this patient.

The other nerve transfers described are not used for ulnar nerve injury. The radial to axillary transfer is a well-described transfer for restoration of deltoid function and shoulder abduction, and it would not be appropriate for this patient with an ulnar nerve injury. The musculocutaneous to anterior interosseous is not a described transfer, although for patients with high median nerve injury, radial nerve branches supplying the brachialis can be used to reinnervate the anterior interosseous nerve. The spinal accessory nerve can be transferred to the suprascapular nerve to reinnervate the supraspinatous and infraspinatous muscles to help restore external rotation for patients with brachial plexus injury. Lastly, the median to posterior interosseous nerve transfer uses the median nerve to restore radial nerve function and would not be appropriate for this patient.

2023

76
Q

An 18-year-old man presents with severe weakness and sensory loss from his right shoulder to his hand. He was involved in a motorcycle collision 3 months ago. Physical examination shows scapular winging. X-ray study of the chest shows that the hemidiaphragm is raised. Electrodiagnostic study demonstrates preservation of sensory nerve action potentials. On the basis of the image shown, which of the following (A–E) is the most likely site of injury in this patient?

A

The correct response is Option E.

The ventral root (motor fibers) and dorsal root (sensory fibers) from the spinal cord converge to form each spinal nerve, which travel through the interscalene space between the anterior and middle scalene muscles. The dorsal roots carry sensory signals back to the spinal cord while the ventral roots carry motor signals to the muscles. The brachial plexus forms from the anterior primary rami of the C5-T1 spinal nerves. The brachial plexus provides motor innervation to the shoulder, including all chest muscles providing control for the glenohumeral joint, motor innervation to the upper extremity, and sensory innervation of the upper limb, except for a small area of the medial upper arm.

At the level of the anterior primary ramus, the nerve branch to the scalene muscles (C5-C8), the nerve branch to the longus colli muscle (C5-C8), the long thoracic nerve (C5-C7), a component of the phrenic nerve (C5 contribution), and component of the dorsal scapular nerve (C5 contribution) all arise.

Subsequent to the interscalene space, the spinal nerves converge to form the three trunks. Each trunk divides into anterior and posterior divisions under the clavicle. Below the clavicle, the anterior and posterior divisions intermix to form the cords: lateral, posterior, and medial. Their nomenclature is based on their position in relationship to the axillary artery. Intermixing the terminal branches of the cords leads to the peripheral nerves: median (from medial and lateral cords), musculocutaneous (lateral cord), ulnar (medial cord), axillary, and radial (posterior cord).

This patient has a Level I injury, which is a preganglionic root injury involving the spinal cord, rootlet, and root. Level I injuries occur in 70% of patients with brachial plexus injuries. Diaphragmatic elevation implies a C4-C5 injury at the level of the root/anterior ramus. Scapular winging implies injury at the level of the root/anterior ramus.

Sensory nerve action potentials (SNAPs) help determine whether a lesion is preganglionic or postganglionic. SNAPs will be preserved in lesions proximal to the dorsal root ganglia (DRG), which house the sensory nerve cell bodies. Sensory input from the periphery runs along sensory fibers and utilize the DRG to reach the brain, bypassing a preganglionic lesion. With preganglionic injuries, SNAPs are normal in an otherwise insensate dermatome and muscle action potentials are absent. SNAPs will be absent in a postganglionic lesion or in the combined pre- and postganglionic lesion.

2023

77
Q

A 54-year-old woman undergoes lipoabdominoplasty under general anesthesia. She is positioned supine with arms abducted for 4 hours. Compression neuropathy of which of the following nerves is most likely in this patient?

A) Long thoracic
B) Median
C) Musculocutaneous
D) Radial
E) Ulnar

A

The correct response is Option E.

Eighty percent of surgical procedures take place supine. The most common postoperative neuropathy following these procedures is ulnar (28% of closed claims in 1999) followed by brachial plexus (20% of closed claims in 1999). They most commonly arise from improper padding and positioning.

Compressive neuropathy of the radial nerve, long thoracic nerve (with its location on the chest wall), musculocutaneous nerve, and median nerve is less common as a result of inadequate positioning or padding during surgery.

Nerve injury may be avoided by abducting the arms no more than 60 to 90 degrees, maintaining supination when arms are abducted, maintaining neutral positioning when arms are tucked at the patient’s side, proper padding on the arm board, and the surgeon not leaning on the extremities throughout the case.

2023